Сохранен 511
https://2ch.hk/sci/res/314306.html
24 декабря Архивач восстановлен после серьёзной аварии. К сожалению, значительная часть сохранённых изображений и видео была потеряна. Подробности случившегося. Мы призываем всех неравнодушных помочь нам с восстановлением утраченного контента!

Математика, тред 35

 Аноним 28/11/15 Суб 01:48:18 #1 №314306 
14486644984540.png
Теоретико-множественный формализм топологических пространств — это исторический курьёз, и формализм локалей подходит для тех же целей гораздо лучше. Например, при его использовании мы автоматически получаем эквивариантные и семейственные версии соответствующих теорем (Хана-Банаха, Гельфанда-Наймарка, Банаха-Алаоглу и других). Как побочный эффект, отпадает всякая необходимость в аксиоме выбора, лемме Цорна и им подобных.

Области применения включают в себя общую топологию, теорию меры, коммутативную алгебру, алгебраическую геометрию, алгебраическую топологию, топологические векторные пространства, операторные алгебры и многие другие.

Предыдущий: >>311874 (OP)
Архивы Аноним 28/11/15 Суб 01:50:14 #2 №314307 
21. http://arhivach.org/thread/93067/
22. https://arhivach.org/thread/94240/
23. https://arhivach.org/thread/95680/
24. https://arhivach.org/thread/96720/
25. https://arhivach.org/thread/99481/
26. https://arhivach.org/thread/100880/
27. https://arhivach.org/thread/101335/
28. http://arhivach.org/thread/106743/
29. http://arhivach.org/thread/109198/
30. http://arhivach.org/thread/114111/
31. https://arhivach.org/thread/116099/
32. https://arhivach.org/thread/118093/
33v1. https://arhivach.org/thread/122613/
33v2. https://arhivach.org/thread/122615/
34. https://arhivach.org/thread/123717/
Аноним 28/11/15 Суб 02:14:45 #3 №314310 
Что такое формализм локалей?
Аноним 28/11/15 Суб 03:24:47 #4 №314313 
Ребят, я, видимо, слегка туповат. Читаю теорию групп, изложенную для умственно отсталых. Пока понятия группы автор еще даже не ввел. Рассматривается пример поворота равностороннего треугольника вокруг центра. Поворачивают его всегда на угол, кратный 120 градусам. Обозначения: а1 - поворот на 120, а2 - на 240, а0 - нулевой поворот или поворот на угол, кратный 360. Далее автор заявляет, что В СООТВЕТСТВИИ СО ЗДРАВЫМ СМЫСЛОМ поворот на 120 градусов обозначим как а0а1.

Вот это а0
а1 мне покоя не дает. Почему "умножить", а не "сложить"? Ведь к изначальному положению мы прибавляем 120 градусов, а не домножаем! Объясните, пожалуйста.
Аноним 28/11/15 Суб 03:55:25 #5 №314316 
>>314313
Это всего лишь обозначение бинарной операции. В данном случае t : X • X -> X, где X -- множество «поворотов», то есть { a0, a1, a2 }. Двум поворотам ставится в соответствие третий, например t(a0, a1) = a1, ибо можно считать значением t поворот, получающийся последовательным применением поворотов-аргументов. Можешь обозначить t звёздочкой/плюсом/кружком и записывать в инфиксной форме. Суть не в этом, а в том, что есть какая-то операция. На то, что у тебя градусы прибавляются, а не домнажаются, также смотреть не надо. В данном случае ты работаешь уже не с градусами (числами), а с некоторыми абстрактными объектами-поворотами, хотя, как я сказал выше, можешь обозначить операцию плюсом и складывать повороты, суть от этого не изменится.
Аноним 28/11/15 Суб 04:00:19 #6 №314317 
>>314313
Умножение или сложение это всего лишь обозначение операции в группе, значок. Можно рисовать звёздочку между аргументами, можно точку, можно плюсик, можно маленький хуец, а можно вообще ничего не рисовать. Обычно сложением обозначают операцию в абелевой (т.е. коммутативной, от перемены мест операций результат не меняется) группе, группа преобразований чего-либо как правило не абелева (матрицы, если ты знаешь что это такое, например, перемножаются некоммутативно), поэтому здесь используется обозначение умножения, а не плюсик сложения.
Аноним 28/11/15 Суб 05:04:15 #7 №314320 
14486762559550.png
>>314316
>>314317

Спасибо, друзья! Суть понял. Хотя в первом посте для меня сложновато написано. А во втором даже слишком просто, даже обидно малость :3 .

Еще нагуглил в вики определение бинарной операции, и немного прихренел. Поясните, пожалуйста, подчеркнутое.
Аноним 28/11/15 Суб 05:44:11 #8 №314321 
>>314320
Понимание постепенно придёт. Возможно, тебе стоит уделить некоторое внимание теории множеств и определению отображения (в Зориче можешь глянуть). Ты должен чётко понимать, с какими объектами работаешь и какие отображения рассматриваешь. В том посте твоя ошибка была в том, что ты отождествил объект-«поворот» и число-«градус». Да, между ними есть некоторая связь, но это разные объекты. Хотя, наверно, об этом рано говорить, если ты только начал читать учебник.

>Поясните, пожалуйста, подчеркнутое.
По определению. Напомню. Если A = B = C, операция внутренняя. Если A = C или B = C, операция внешняя. Из того, что A = B = C, следует, что A = C так же, как следует, что B = C. Следовательно, свойства внутренней операции влекут свойства внешней, то есть любая внутрення операция является внешней. Возможно, это покажется нелогичным, но можешь принять это как игру слов/определений. Можешь пока не думать об этом.
Аноним 28/11/15 Суб 05:57:59 #9 №314322 
>>314320
Пройди тест http://psyfactor.org/tests/intro.php?dir=logic
Аноним 28/11/15 Суб 06:43:39 #10 №314324 
14486822199990.png
>>314306 (OP)
Научись рассказывать не только о достоинствах, но и о недостатках задвигаемой теории, мгазь.
Аноним 28/11/15 Суб 07:51:17 #11 №314328 
>>314324
Это вербит написал, а не оп.
Аноним 28/11/15 Суб 10:35:01 #12 №314334 
>>314310
https://en.wikipedia.org/wiki/Complete_Heyting_algebra

Алгебра Гейтинга H - это ограниченная решетка (то есть существуют 0 и 1), такая, что для всех a, b ∈ H существует наибольший элемент x ∈ H такой, что a∧x ≤ b. Этот элемент называется относительным псевдо-дополнением к a и обозначается a → b.

Полная алгебра Гейтинга - это алгебра Гейтинга, которая полна как решётка.

Множество всех открытых множеств топологического пространства, упорядоченное по включению, образует полную алгебру Гейтинга.
Множество всех подмножеств данного множества, упорядоченное по включению, образует полную алгебру Гейтинга.

Полные алгебры Гейтинга являются объектами трёх разных категорий: категории CHey, категории локалей Loc, категории фреймов Frm.

Существует функтор O из категории топологических пространств Top в категорию локалей. Любая локаль имеет топологическое пространство, наилучшим образом её приближающее, оно называется спектром локали. Существует функтор S из Loc в Top, служащий правым сопряжённым функтором для O.

На русском языке об этом нет ни одного документа.
sageАноним 28/11/15 Суб 11:05:23 #13 №314337 
>>314334
>На русском языке об этом нет ни одного документа.
О чем? Например в "Теории решеток" Биркгофа есть глава о полных решетках в которых рассматриваются полные брауэровы решётки, что есть суть теже гейтинговы алгебры.
Аноним 28/11/15 Суб 11:22:55 #14 №314340 
>>314334
>На русском языке об этом нет ни одного документа.
Этот западнец порвался, несите следующего.
Аноним 28/11/15 Суб 11:37:19 #15 №314345 
>>314337
О локалях и фреймах, вестимо.
Аноним 28/11/15 Суб 13:42:01 #16 №314360 
>>314306 (OP)

Формализм аксиом Пеано — это исторический курьёз.
Аноним 28/11/15 Суб 14:06:25 #17 №314367 
Поясните за терминологию. Я так понял inner product space по-русски никак не называется?
Аноним 28/11/15 Суб 14:16:55 #18 №314370 
>>314367
Оно называется "эрмитово пространство".
Аноним 28/11/15 Суб 15:51:29 #19 №314399 
>>314313
> Читаю теорию групп, изложенную для умственно отсталых
ссылку?
Аноним 28/11/15 Суб 16:08:39 #20 №314408 
>>314306 (OP)
Читаю Киселева и ничего не понимаю, может быть существуют учебники полегче, совсем для ньюфагов? Начал знакомство с матаметикой с учебника арифметики.
Аноним 28/11/15 Суб 16:24:14 #21 №314411 
14487170544120.png
>>314408

Аноним 28/11/15 Суб 17:04:55 #22 №314425 
>>314411
лол. Я про математика из Петербурга.
Аноним 28/11/15 Суб 19:06:04 #23 №314444 
Пусть T - линейный оператор на конечномерном ненулевом комплексном векторном пространстве. Верно ли, что T имеет ненулевое собственное значение тогда и только тогда, когда T != 0?
Аноним 28/11/15 Суб 19:13:53 #24 №314447 
>>314444
А что такое собственное значение нулевого оператора?
Аноним 28/11/15 Суб 19:15:55 #25 №314449 
>>314444
Я думаю, что да.

Пусть T = 0. Тогда, очевидно, у него нет собственных значений кроме нуля.

Пусть у T нет собственных значений кроме 0. Существует базис V из собственных векторов T. Назовем его v_1, ..., v_n. Так как для любого j: T(v_j) = 0, T = 0.
Аноним 28/11/15 Суб 19:16:32 #26 №314450 
>>314447
0.

Это я сам себе ответил, если че >>314449
Аноним 28/11/15 Суб 19:17:08 #27 №314451 
>>314447
0.

Это я сам себе ответил, если че >>314449
Аноним 28/11/15 Суб 19:29:59 #28 №314454 
14487281997600.png
>>314444
Просто я придумал свое доказательство пикрелейтед, потом посмотрел то, что в учебнике, а оно какое-то абсолютно упоротое.

Вот мое. Пусть (a, v) - собственная пара T. Тогда
a||v||2 = <av, v> = <Tv, v> = 0.
То есть a = 0. Так как любое собственное число T равно 0, T = 0.
Аноним 28/11/15 Суб 19:50:15 #29 №314460 
>>314444
Очевидно, нет. Возьми любой ненулевой нильпотентный оператор.
Аноним 28/11/15 Суб 20:00:21 #30 №314463 
Взял
[0 0
1 0]
Нашел ошибку у себя
> Существует базис V из собственных векторов T
Аноним 28/11/15 Суб 21:34:44 #31 №314501 
Где найти решения примеров из Зорича?
Аноним 29/11/15 Вск 02:18:35 #32 №314542 
>>314501
Решить самому?
Аноним 29/11/15 Вск 02:37:22 #33 №314546 
Есть ли какой-нибудь список литературы упорядоченный для учебной программы Вербитского начиная с самого первого пункта.

http://imperium.lenin.ru/~verbit/MATH/programma.html
Тут список дается, начиная с первого курса только.
Аноним 29/11/15 Вск 03:51:36 #34 №314558 
>>314546
http://ium.mccme.ru/f04/experimental.html
http://rghost.ru/6cQMDZxSD - перегнанный в pdf
Аноним 29/11/15 Вск 06:00:57 #35 №314567 
Кто нибудь тут способен опровергнуть ...999 = -1? Конкретно, показать, что у этого утверждения есть противоречивые следствия.
Аноним 29/11/15 Вск 07:03:52 #36 №314570 
>>314567
Пусть ...999 = -1. Пришли к противоречию.
Аноним 29/11/15 Вск 07:43:55 #37 №314571 
Математикопетухи, а вот хоть одна ваша теоремка помогла реально людям? Может, накормила миллионы? Например, говно, которое перельман доказал аж за лям. Нахуя оно всралось?
Аноним 29/11/15 Вск 08:30:01 #38 №314574 
>>314571
Утилитаристы не нужны. Благодаря математике ты видишь сегодняшний мир таким как он есть, этого достаточно.
Аноним 29/11/15 Вск 09:07:30 #39 №314577 
>>314570
0.(9) = 1 - пришли к противоречивую. Так по твоему. Давай без мозгоебства. Мне действительно интересен вразумительное доказательство того, что это не верно.
Аноним 29/11/15 Вск 09:09:22 #40 №314578 
>>314571
Кто тебе сказал, что кто-то кому то что-то должен? Ты не прихуел? Каждый делает лишь то, что хочет.
Аноним 29/11/15 Вск 09:19:28 #41 №314580 
>>314570
Ты вопрос задай нормально. Что ты имеешь ввиду под 99999... Сумму бесконечного ряда? так в классическом смысле она не сходится. Определи правило суммирования, тогда можно будет о чём-то говорить.
Аноним 29/11/15 Вск 09:20:22 #42 №314581 
>>314580
Сюда >>314567
Аноним 29/11/15 Вск 09:27:39 #43 №314582 
>>314580
Я иммею ввиду лишь то, что происходит в этом видео. Не больше и не меньше https://www.youtube.com/watch?v=x-fUDqXlmHM
Аноним OP 29/11/15 Вск 09:29:39 #44 №314583 
14487785790650.gif
>>314582
Ох нет. Это же спор 0.(9) = 1.
Аноним 29/11/15 Вск 10:14:47 #45 №314584 
>>314583
Нет, вопрос все тот же >>314567
Аноним 29/11/15 Вск 12:10:12 #46 №314606 
14487882128810.png
>>314582
Ты можешь просто съебать? Тебе уже 100 раз объянили, что нет такого объекта как ...9999. Если ты можешь его определить так, чтобы он имел смысл и сможешь доказать, что с ним можно делать операции, которые делаются в этом видео, то напиши. Если не можешь, то иди на хуй.
Аноним 29/11/15 Вск 13:09:47 #47 №314628 
>>314580
>...999
Может это он про p-адические числа?
Аноним 29/11/15 Вск 15:04:22 #48 №314685 
Пацаны, почему авторы учебников всегда дают материал как-то вширь? Почему не дают только ключевые результаты, которые повсеместно используются и леммы необходимые для их доказательства?
Аноним 29/11/15 Вск 15:05:35 #49 №314687 
>>314685
>ключевые результаты, которые повсеместно используются
Такой вещи не существует. Каждый результат самоценен и используется в лучшем случае в двух-трёх местах.
Аноним 29/11/15 Вск 15:18:10 #50 №314692 
Привет, Аноны. Сдаю академическую разницу, экз по линейной алгебре и тут возник вопрос: подскажите пожалуйста, что из себя представляет сам по себе линейный оператор? Что подразумевается под "Оператор А, действующий из V в W"? Что значит действующий ИЗ-В?
Аноним 29/11/15 Вск 15:22:00 #51 №314694 
>>314692
>Что подразумевается под "Оператор А, действующий из V в W"
Берешь элемент из векторного пространства V, предположим v, действуешь на него оператором A, получаешь элемент Av, который принадлежит векторноиу пространству W
Аноним 29/11/15 Вск 15:32:57 #52 №314701 
>>314694
Спасибо!
Аноним 29/11/15 Вск 15:34:05 #53 №314702 
14488004452640.jpg
>>314701
Всегда пожалуйста!
Аноним 29/11/15 Вск 15:38:02 #54 №314703 
>>314567
А что тебе не нравится в этом утверждении, и с чем должно противоречить? Оно получается как предел по норме |a+1|_10=0, соответственно, проблемы могут возникнуть только с нормой, а не с алгебраическими операциями. Возникающие проблемы - неоднозначность разложения и отсутствие правильного деления.
Аноним 29/11/15 Вск 16:12:45 #55 №314705 
>>314687
Вот, например, в учебнике есть утверждение о том, что у самосопряженного оператора все собственные значения вещественные. Во-первых, на это утверждение не ссылаются последующие утверждения. Во-вторых, его можно доказать самому через минуту размышлений. Зачем надо было его включать? Мне не жалко, просто если такие вещи убрать, то можно сократить материал раза в 2-3 без потери глубины. И такое я замечал во всех учебниках.
Аноним 29/11/15 Вск 18:24:30 #56 №314727 
>>314705
В нормальных учебниках подобное выносится в упражнения.
Аноним 29/11/15 Вск 18:41:57 #57 №314729 
>>314628
Тебя уже обоссали в прошлом треде с твоими p-адическими числами, за которые ты не смог пояснить по понятиям. Что такое 0.9999... в р-адических, уёба?
Аноним 29/11/15 Вск 18:43:43 #58 №314730 
>>314584
А давай так, пока мы тебе всем тредом в рот не нассали ты даёшь определение того, что такое "вещественное число", потом того, что такое "...999", а потом уже задаёшь свои невероятно тупые вопросы.
Аноним 29/11/15 Вск 18:50:22 #59 №314732 
>>314703
А давай так, пока мы тебе всем тредом в рот не нассали ты даёшь определение того, что такое "п-адическое число", потом того, что такое "0.999..." в п-адических числах, а потом уже делаешь свои невероятно тупые умозаключения.
Аноним 29/11/15 Вск 18:52:05 #60 №314734 
>>314730
>>314732
Может сразу нассать ему(им) в рот?
Аноним 29/11/15 Вск 19:42:17 #61 №314741 
>>314571
> Вот эта твоя гайка, петрович, которую ты завернул когда капот монтировал, она людям что-нибудь дала? Она спасла мир от войн, накормила миллионы? НЕ НУЖНО!
Аноним 29/11/15 Вск 19:44:40 #62 №314742 
>>314606
В чём когомологический эффект? Типа все мемы /sci/ скомбинированы?
Аноним 29/11/15 Вск 19:54:21 #63 №314744 
14488160614640.png
>>314742
Если шутку объяснить, она не будет смешной.
Аноним 29/11/15 Вск 20:10:22 #64 №314745 
14488170228730.png
Решаю Munkres Topology. Поясните за пикрелейтед. Я решил 3а, а в 2 и 3б вообще нет никаких идей. Мне кажется, автор создает искусственную сложность, подбирая неебические задачи, потому что материал слишком простой для понимания. Или это элементарно, а я просто туплю? или тупой
Аноним 29/11/15 Вск 20:13:35 #65 №314746 
>>314745
2 придумал как решить, пока пост писал. Не знаю, зачем вообще писал.
Аноним 29/11/15 Вск 20:31:42 #66 №314747 
>>314745
>3a) Показать, что метрика есть непрерывная функция на метрическом пространстве с этой же метрикой.
>искусственную сложность, неебические задачи
>материал слишком простой для понимания.
HA-HA-HA, oh you.
Аноним 29/11/15 Вск 20:35:04 #67 №314748 
>>314747
Ну это я как раз решил, если ты читал.
Аноним 29/11/15 Вск 20:48:17 #68 №314749 
>>314746
>>314745
Если кому интересно, решение 2 следующее.
D((x1, y1), (x2, y2)) = 1, если x1 != x2. Иначе D((x1, y1), (x2, y2)) = min(|y1 - y2|, 1).
knuebok 29/11/15 Вск 21:06:07 #69 №314754 
>>314749
3б) Нужно по сути показать, что: тождественное отображение Id : (X',d') -> (X,d) непрерывно <-> топология (X',d') тоньше, чем (X,d). Однако это проверяется тупо по определению, прообраз любого открытого множества в (X,d) относительно Id открыт в (X',d'), значит, в (X',d') открытых множеств, как минимум, не меньше, чем в (X,d).
Аноним 29/11/15 Вск 21:10:30 #70 №314756 
14488206307320.png
Как найти?
Чисто гипотетически, из википедии, я нашел ответ, но алгебраически я туплю.
Для второго надо ведь просто завести дельту под степень, разложить степенной двучлен и потом просто отнять исходную функию и сокртить все что с дельтой, чтобы получить 4X^3.
Для третьего что делать? Засунуть дельту в скобки и разложить как квадратный трехчлен? Или засунуть изначальную функцию в какую то букву и решать так же как в первом случае?
Четвертый и пятый вроде все просто, надо только в конце свести дробь к общему знаменателю.
А к шестому я подсказку не понял.
knuebok 29/11/15 Вск 21:14:11 #71 №314757 
>>314756
Раскрыть скобки, взять предел. Пределы отношения многочленов брать не учили, что ли?
>А к шестому я подсказку не понял.
Она довольно недвузначная. Умножь на ту самую сумму числитель и знаменатель, раскрой скобки, посмотри что выйдет.
Аноним 29/11/15 Вск 21:16:29 #72 №314759 
>>314757
>Пределы отношения многочленов
Не знаю что это такое, так как самообучение.
Аноним 29/11/15 Вск 21:18:03 #73 №314760 
>>314744
Ты чё ты чё..
knuebok 29/11/15 Вск 21:18:27 #74 №314761 
>>314759
У тебя странное самообучение. Понятие "производная" - более сложное, чем понятие "предел", так как опирается на него (по крайней мере в классическом подходе). А так выходит что ты пытаешься найти "не знаю что, не зная как".
Аноним 29/11/15 Вск 21:19:53 #75 №314762 
>>314761
Я про отношение многочленов. Тобишь что это и как его и куда.
Предел я более менее усвоил, отношение функции с малым приращением к самой функции.
knuebok 29/11/15 Вск 21:21:51 #76 №314764 
>>314762
>Предел я более менее усвоил, отношение функции с малым приращением к самой функции.
Это как раз производная.

Раскрой всевозможные скобки в числителе выражения (x+1+delta)^2-(x+1)^2/delta и возьми предел delta->0..
Аноним 29/11/15 Вск 21:23:26 #77 №314765 
>>314749
Задачи на метризуемость того или иного пространства это по большей части задачи на использование одноимённого критерия https://ru.wikipedia.org/wiki/%D0%9C%D0%B5%D1%82%D1%80%D0%B8%D0%B7%D1%83%D0%B5%D0%BC%D0%BE%D0%B5_%D0%BF%D1%80%D0%BE%D1%81%D1%82%D1%80%D0%B0%D0%BD%D1%81%D1%82%D0%B2%D0%BE . Не обязательно нужно эту метрику руками строить.
knuebok 29/11/15 Вск 21:26:04 #78 №314767 
>>314765
А почему RxR с лексикографическим порядком - нормальное пространство? Или ты не про теорему Тихонова о метризуемости?
Аноним 29/11/15 Вск 21:27:03 #79 №314768 
>>314764
>(x+1+delta)^2
Вот это как разложить? Обычный квадрат сумму я знаю а^2+/-2аb+b^2
knuebok 29/11/15 Вск 21:28:51 #80 №314769 
>>314768
((x+1)+delta)^2 например. Можешь даже замену сделать a=x+1 и b=delta, если тебе так удобнее.
Аноним 29/11/15 Вск 21:33:31 #81 №314770 
>>314767
Ну, это легко доказать, в этом и состоит упражнение.
Аноним 29/11/15 Вск 21:40:38 #82 №314772 
>>314769
Потом еще раз открывать а после всех делов?
А то у меня получилась херня какая то.
(x+1)^2+2Δx+Δx^2-(x+1)^2/Δx=2Δx+Δx^2/Δx=~0
Аноним 29/11/15 Вск 21:44:47 #83 №314773 
>>314772
А если раскрыть вообще все скобки выйдет 4х+2
knuebok 29/11/15 Вск 21:45:22 #84 №314774 
>>314772
Ты неправильно раскрыл, перепроверь.
>>314770
Докажи, если легко. Судя по логике книги, упражнение как раз в том, чтобы привести явную метрику, так как к моменту этого упражнения ещё не рассказывают о теоремах метризуемости.
Аноним 29/11/15 Вск 21:47:33 #85 №314775 
>>314774
Я сперва подставил буквы, раскрыл с буквами и потом вместо буковок поставил исходные цифры. Вроде все правильно сделал.
knuebok 29/11/15 Вск 21:49:39 #86 №314776 
>>314775
Вроде нет. Должно было выйти (x+1)^2 + 2 delta (x+1) + delta^2 - (x+1)^2 / delta
Аноним 29/11/15 Вск 21:52:07 #87 №314777 
>>314776
Ага, вижу ошибку свою.
Но ведь потом две дельты икс плюс один все равно становятся крайне малы и остается только квадрат минус квадрат. Их тоже раскрывать?
Аноним 29/11/15 Вск 21:54:10 #88 №314778 
>>314765
Ну я до этого еще не дошел. Это только страниц через 100 будет.
knuebok 29/11/15 Вск 22:07:11 #89 №314781 
>>314777
Ты сначала подели на дельта, а потом подставляй 0.
Аноним 29/11/15 Вск 22:11:29 #90 №314786 
>>314781
Распиши мне как для дауна, а то я не понимаю.
Результат должен быть (x+1)^2 + 2 (x+1) + delta - (x+1)^2?
knuebok 29/11/15 Вск 22:14:08 #91 №314787 
>>314786
[ (x+1+delta)^2-(x+1)^2 ]/delta =
[ (x+1)^2 + 2 (x+1) delta + delta^2 - (x+1)^2 ]/delta =
[ 2 (x+1) delta + delta^2]/delta =
[ 2 (x+1) + delta]
при delta -> 0 получаем просто 2 (x+1).
А вообще с такими вопросами в образовач (un), а не в саентач: тут на школьные дз агрятся.
Аноним 29/11/15 Вск 22:16:37 #92 №314788 
>>314787
Это в какой такой школе пределы и производные проходят?
Да и в ун сидят три с половиной долбоеба которым интереснее обсуждать кто в какуй универ поступит чтобы соснуть меньше чем другие.
knuebok 29/11/15 Вск 22:18:10 #93 №314789 
>Это в какой такой школе пределы и производные проходят?
Да во всех, вроде, в 10-11 классах есть предмет такой: "алгебра и начала анализа", вот в "началах анализа" и проходят. Хотя я не знаю как оно на практике, сам учился на физ.мате, а не в обычной школе.
Аноним 29/11/15 Вск 22:19:49 #94 №314790 
>>314789
В моей школе такого нет. Только матрицы и связанные с ними примеры раздрачиваем.
knuebok 29/11/15 Вск 22:39:51 #95 №314794 
>>314765
Кстати, вполне очевидно, что RxR с лексикографическим порядком - не пространство со счётной базой. В частности, множества вида
U_x0 = {(x0,y) \in R^2 : 0<y<1}
образуют несчётную систему непересекающихся открытых множеств. Поэтому теорема Урысона неприменима.
Аноним 29/11/15 Вск 22:40:40 #96 №314795 
>>314789
Производные дают везде. Это материал ЕГЭ.
Аноним 29/11/15 Вск 22:41:54 #97 №314796 
>>314795
Дякую тобi, боже, що я не москаль.
Смекаешь?
Аноним 29/11/15 Вск 22:50:56 #98 №314798 
>>314754
Не понял твоего решения. Что такое d'? Что такое топология (X',d')?
knuebok 29/11/15 Вск 22:56:36 #99 №314802 
>>314798
Ну там же в условии дано что, Х' - метрическое пространство. Или не обязательно метрическое? Из контекста непонятно.
Аноним 29/11/15 Вск 23:04:34 #100 №314805 
>>314802
Не, в условии написано, что X без штриха - метрическое пространство.
knuebok 29/11/15 Вск 23:05:42 #101 №314807 
>>314805
А Х со штрихом это что?
Аноним 29/11/15 Вск 23:06:20 #102 №314808 
14488275806580.png
>>314805
>>314802
Более того, вот что там написано на следующей странице. То есть X' - произвольное пространство.
Аноним 29/11/15 Вск 23:26:29 #103 №314812 
14488287896020.png
>>314794
Чого ти несёшь? Открытое множество a<x<b в лексикографическом порядке выглядит как пикрелейтед. Всё тут со счётной базой, всё отделяется, всё регулярно.
knuebok 29/11/15 Вск 23:27:21 #104 №314813 
>>314812
Предъяви счётную базу.
Аноним 29/11/15 Вск 23:29:47 #105 №314814 
>>314813
За щекой проверь.
Аноним 29/11/15 Вск 23:30:48 #106 №314815 
>>314813
Множества с рациональными a и b.
knuebok 29/11/15 Вск 23:32:30 #107 №314816 
>>314815
Это не база. Например, нельзя представить множество {(pi,x) : 0<x<1} в виде объединения элементов базы.
Аноним 29/11/15 Вск 23:36:09 #108 №314817 
>>314816
Оно приближается множествами {(an,x), 0<x<1} где an - последовательность рациональных, сходящаяся к pi.
knuebok 29/11/15 Вск 23:41:21 #109 №314819 
>>314817
Что значит "приближается"? База - это когда любое открытое множество в виде объединения представить можно, покажи мне семейство множеств из твоей базы, объединение которых даст {(pi,x), 0<x<1}.
Аноним 29/11/15 Вск 23:41:43 #110 №314820 
Ты сказал, что не будешь.
Аноним 29/11/15 Вск 23:42:53 #111 №314821 
>>314817
Или нет, немного не то написал, в моих обозначениях это будут множества c a_n=(x_n, 0), b_n = (y_n, 1) где x_n и y_n сходящиеся к пи последовательности снизу и сверху соответственно.
knuebok 29/11/15 Вск 23:48:02 #112 №314823 
>>314821
Между (2/3,0) и (1,1) будет элемент (3/4,12482384234), например, ты это осознаешь?
Аноним 29/11/15 Вск 23:48:18 #113 №314824 
14488300986670.png
>>314802
Короче, вот че я придумал. Буду признателен, если проверишь на предмет ошибок.

Будем обозначать открытый шар в метрике $d$ с центром $x$ радиуса $\varepsilon$ как $B_d(x, \varepsilon)$. Надо показать, что для любого $x$ и любого шара $B_d(o, \varepsilon)$, содержащего $x$, найдется базисный элемент $X'$ $U$ такой, что $x \in U$ и $U \subset B_d(x, \varepsilon)$. Выберем любой $x_0 \in X$ и шар, в котором он содержится. Без потери общности можно считать, что $x_0$ - центр шара (если нет, можно взять шар с центром в $x_0$, целиком содержащийся в большем шаре, и найти нужный базисный элемент для него). Пусть радиус шара равен $\varepsilon$.

Выберем $y_0 \in B_d(x_0, \varepsilon/3)$. Определим $d_{y_0}: X' \to \mathbb{R}$ как $d_{y_0}(x) = d(x, y_0)$. Я знаю, что если функция непрерывна, то она непрерывна по каждой координате, то есть $d_{y_0}$ непрерывна. Поэтому можно выбрать базисный элемент $X'$ $U$ такой, что $x_0 \in U$ и $d_{y_0}(U) \subset B_d(d_{y_0}(x_0), \varepsilon/3)$ (по теореме 18.1(d), пикрейлейтед), то есть $\forall u \in U: d(x_0, y_0) - \varepsilon/3 < d(u, y_0) < d(x_0, y_0) + \varepsilon/3$.

По неравенству треугольника
$$d(x_0, u) \leq d(x_0, y_0) + d(y_0, u) < d(x_0, y_0) + d(x_0, y_0) + \varepsilon/3 < \varepsilon/3 + \varepsilon/3 + \varepsilon/3 = \varepsilon.$$
То есть, мы показали, что $U \subset B_d(x_0, \varepsilon)$.
Аноним 29/11/15 Вск 23:51:49 #114 №314826 
>>314821
Хотя да, именно такие множества и не получится представить.
Аноним 30/11/15 Пнд 00:07:51 #115 №314828 
>>314824
Я какую-то хуйню написал, начиная с выбора $y_0$ (там еще ошибка в том, что не факт, что такое $y_0$ можно выбрать). Лучше вот так.

Определим $d_{x_0}: X' \to \mathbb{R}$ как $d_{x_0}(x) = d(x, x_0)$. $d_{x_0}$ непрерывна. Выберем базисный элемент $X'$ $U$ такой, что $x_0 \in U$ и $d_{x_0}(U) \subset B(d_{x_0}(x_0), \varepsilon) = B(0, \varepsilon)$. Это можно сделать по (18.1). Таким образом, $\forall u \in U: d(x_0, u) < \varepsilon$, т.е. $U \subset B_d(x_0, \varepsilon)$.
Аноним 30/11/15 Пнд 00:09:49 #116 №314829 
>>314558
Там задания как я понял для поступления. Энивей нашел литературу в каком-то блоге.
Мало ли кто ещё искать будет
http://heller.ru/blog/2010/04/ium-in-details/
Аноним 30/11/15 Пнд 00:20:19 #117 №314833 
>>314732
Тебе, тупому дауну, уже на пальцах все показали, а до тебя даже определение дойти не может. Если ты и способен кого то обоссать, то только себя и свою мамку.
Аноним 30/11/15 Пнд 00:26:59 #118 №314835 
14488324196420.png
>>314833
Ты понимаешь, что тебе уже всем тредом в ротешник ссут?
Аноним 30/11/15 Пнд 00:41:37 #119 №314836 
>>314835
Успокойся уже ущербный. Просто признай свою умственную неполноценность и не мешай людям общаться.
Аноним 30/11/15 Пнд 00:42:54 #120 №314837 
>>314836
Я не он >>314732
Я же, говорю, всем тредом ссут, а ты не веришь.
Аноним 30/11/15 Пнд 02:10:27 #121 №314846 
>>314837
Раздвоение личности? Может тебе стоит в /sn/ пройти, поехавший?
knuebok 30/11/15 Пнд 03:37:31 #122 №314848 
>>314828
Сорян, сейчас прочту.
knuebok 30/11/15 Пнд 03:42:59 #123 №314849 
>>314846
И таки в p-адических числах нету чисел с "бесконечной дробной частью".
Аноним 30/11/15 Пнд 06:55:19 #124 №314864 
>>314828
Да, всё верно.
Аноним 30/11/15 Пнд 11:16:38 #125 №314900 
N-даун поменял профиль и теперь он п-даун? Одобряю.
Аноним 30/11/15 Пнд 11:55:30 #126 №314913 
>>314900

Выебал анально твою мамку-шлюху. Моё почтение. Одобряю.
Аноним 30/11/15 Пнд 12:43:21 #127 №314923 
>>314913
Такие то фантазии.
Пеано давно последний раз снился?
Аноним 30/11/15 Пнд 13:00:15 #128 №314931 
>>314900
Эволюционирует.
Аноним 30/11/15 Пнд 14:23:30 #129 №314958 
>>314923

Это реальность. Не веришь, спроси у своей мамки-шлюхи, она всё подтвердит.
Аноним 30/11/15 Пнд 14:24:20 #130 №314959 
>>314931

Напомню, что никто здесь так и не дал корректное определение N. Так зачем ты пиздишь???
Аноним 30/11/15 Пнд 17:59:37 #131 №314998 
Скиньте ту пасту, где "математик вложил себе 2 пальца в рот. Математик завел натуральные числа через аксиомы Пеано".
Аноним 30/11/15 Пнд 18:03:47 #132 №314999 
14488958273590.jpg
Я не понял. Читаю матан по Фихтенгольцу. Вижу фигу, что дескать 0.(9) есть 1.
Какого хуя? То есть, исходя из этого, можно определённо точно сказать, что каждое целое число - лишь какая-то дробь с бесконечным порядком, которая стремится к вот к целому числу.
Что не так? Где я не прав? Прав ли Фихтенгольц или пидор-защеканец?
Аноним 30/11/15 Пнд 18:25:57 #133 №315005 
14488971576040.jpg
>>314959
Пора напомнить твое место
Аноним 30/11/15 Пнд 18:50:34 #134 №315009 
>>314999
Защеканец немножк.
Аноним 30/11/15 Пнд 18:51:03 #135 №315010 
>>314999
Каждое вещественное число - это класс эквивалентности бесконечных десятичных дробей. Бесконечные десятичные дроби 1.000.... и 0.999... входят в один и тот же класс.

Кстати, ты врёшь, что читаешь Фихтенгольца.
Аноним 30/11/15 Пнд 18:55:04 #136 №315014 
>>314998 -> >>315013
Аноним 30/11/15 Пнд 18:55:24 #137 №315015 
14488989249190.png
>>315010
Нет, не вру. Соси.
Но спасибо, что пояснил.
Аноним 30/11/15 Пнд 19:26:57 #138 №315023 
>>314542
Может всё же есть где-то? Очень нужно, препод чмо, не на что ровняться без решений задач.
Аноним 30/11/15 Пнд 19:35:34 #139 №315027 
>>314999
Лучше пропусти эту главу. Есть более хорошие способы определения R, чем десятичные дроби.

Сейчас на примере корней расскажу, какое свойство R самое важное. У Q этого свойства нет. Возьмем, например, множество рациональных чисел x таких, что x2 < 2. В поле Q у него нет точной верхней грани, потому что корень из 2 - иррациональное число. При этом нам хочется, чтобы оно имело точную верхнюю грань.

Можно построить такое поле R, которое содержит Q в качестве подполя и в котором операции сложения и умножения совпадают с операциями в Q и у любого ограниченного сверху множества есть sup, а у любого ограниченного снизу - inf. То, как строится R, нет смысла читать, потому что построение довольно длинное и нигде тебе не пригодится. С другой стороны, если интересно или если ты не можешь поверить в это свойство (хотя в школе это свойство используют в течение 11 лет), то почитай.

С помощью описанного свойства можно доказать, например, что Q плотно в R (то есть, что между любыми двумя вещественными числами найдется рациональное), что существует корень n-й степени из произвольного положительного числа. Можно определить расстояние на плоскости от точки x до множества A как inf{расстояние от x до a, где a - элемент A}. Мы знаем, что множество {расстояние от x до a, где a - элемент A} ограничено снизу нулем (расстояние всегда неотрицательное) и поэтому имеет точную нижнюю грань.

Что касается десятичных чисел, то их лучше воспринимать просто как форму записи и не задумываться об этом.
> Вижу фигу, что дескать 0.(9) есть 1.
Это правда. Но ниче такого тут нет, потому что это просто форма записи.

> То есть, исходя из этого, можно определённо точно сказать, что каждое целое число - лишь какая-то дробь с бесконечным порядком, которая стремится к вот к целому числу.
Ну можно и так сказать. Например, 1 = 1.(0). Не вижу ниче такого. Но лучше определять вещественные числа как я выше написал.
Аноним 30/11/15 Пнд 19:50:02 #140 №315031 
>>315027
Забыл написать, как воспринимать десятичную форму записи. Десятичную форму записи нужно воспринимать так, что число, которое в этой форме записано, равно тому, что перед запятой + сумма dn10-n, n от 1 до бесконечности, где dn - n-я цифра после запятой.
Что такое сумма бесконечного ряда, потом прочитаешь.
Аноним 30/11/15 Пнд 20:05:06 #141 №315035 
>>315031
Спасибо.
Пойду чертить графики, чтобы понять.
Аноним 30/11/15 Пнд 20:21:06 #142 №315037 
Почему R - подмножество C?

Допустим, мы определили C как множество чисел вида a + bi, где a и b вещественные и i2 = -1. Очевидно, такое определение никуда не годится, потому что непонятно, почему вообще должно существовать такое число i.

Допустим, мы определили C как R2. Как R может быть подмножеством R2?
Аноним 30/11/15 Пнд 20:28:24 #143 №315038 
>>315037
Ну b может быть равно 0
Аноним 30/11/15 Пнд 20:34:54 #144 №315041 
>>315038
То есть ты определяешь C вот так
> множество чисел вида a + bi, где a и b вещественные и i2 = -1
? Откуда ты знаешь, что такое число i существует? Из какого множество берется это i?
Аноним 30/11/15 Пнд 20:36:42 #145 №315042 
>>315041
Открой книжку хорошую и посмотри как определяются комплексные числа через упорядоченные пары, как они умножаются. Оттуда будет следовать существование i.
Аноним 30/11/15 Пнд 20:42:25 #146 №315044 
>>315042
Ну я знаю про это определение.
> Допустим, мы определили C как R^2. Как R может быть подмножеством R^2?
Аноним 30/11/15 Пнд 20:45:04 #147 №315045 
>>315044
Бля, представил как меня называют C-петухом и проиграл в голосяндру.
Аноним 30/11/15 Пнд 20:49:48 #148 №315047 
>>315044
R туда вкладывается изоморфно, т.е R=(0,R)<(R,R). Если занимаешься математикой, такие недоговорки будут сплошь и рядом, ошибок в
этом, привыкай.
Аноним 30/11/15 Пнд 20:50:38 #149 №315048 
>>315045
Темы каждой борды рано или поздно вырождаются в то, о чем удобно спорить.
Аноним 30/11/15 Пнд 20:55:12 #150 №315050 
Я недавно поругался с моим преподавателем матанализа. Он сказал, что категории - бесполезная ерунда, и что он принципиально не желает и не будет их изучать. А кто изучает, тех хочет выгнать из профессии за очковтирательство.
Мне грустно, котаны.
Аноним 30/11/15 Пнд 21:02:29 #151 №315052 
Я недавно поругался с моим преподавателем матанализа. Он сказал, что определение N - бесполезная ерунда, и что он принципиально не желает и не будет его изучать. А кто изучает, тех хочет выгнать из профессии за очковтирательство.
Мне грустно, котаны.
Аноним 30/11/15 Пнд 21:03:33 #152 №315053 
>>315050

Он прав, это всего лишь ссаные стрелочки, без теории множеств и функций это говно даже интуитивной интерпретации не имеет. Баловство одно, короче.
Аноним 30/11/15 Пнд 21:08:55 #153 №315054 
Я недавно поругался с моим преподавателем матанализа. Он сказал, что пределы - бесполезная ерунда, и что он принципиально не хочет их лопиталить и разлагать по формуле Тейлора. А тех кто находит их вручную, хочет выгнать из профессии за пребывание в каменном веке.
Аноним 30/11/15 Пнд 21:09:51 #154 №315055 
>>315052
>>315053
>>315054
Вам смешно, а у меня драма.
Аноним 30/11/15 Пнд 21:24:29 #155 №315060 
>>315047
Ну это понятно. Просто одно дело, когда какую-нибудь статью читаешь и там незначительная ошибка/недоговорка, а другое дело, когда читаешь определение чего-то такого базового и простого как C и в английской и русской википедии такая же недоговорка.
Аноним 30/11/15 Пнд 21:39:33 #156 №315071 
>>314999
>>315010
>>315027
В Зориче, к примеру, доказывается, исходя из построений, что не всякому символу (десятичной дроби) соответствует число. Получается, что 0.(9) — символ, но не число.
Аноним 30/11/15 Пнд 21:46:26 #157 №315073 
>>315071
>>не всякому символу (десятичной дроби) соответствует число
Всякому (если целая часть конечная).
Аноним 30/11/15 Пнд 22:06:54 #158 №315083 
>>315073
По построению Зорича — не всякому.
http://edu.alnam.ru/book_z_math1.php?id=22
(в конце страницы)
В том числе для символа 0.(9) нет соответствующего числа.
Аноним 30/11/15 Пнд 22:09:11 #159 №315085 
>>315083
Ссылка вставилась не так. Нажмёшь внизу 67, окажешься на 61 стр. Там внизу.
Аноним 30/11/15 Пнд 22:11:42 #160 №315087 
>>315083
Плохое построение. Негодное.
Аноним 30/11/15 Пнд 22:15:18 #161 №315090 
>>315087
Можешь лучше? И что в нём плохого?
Аноним 30/11/15 Пнд 22:23:30 #162 №315092 
>>315050
>>315052
>>315054
Ну, он прав. Для матанализа они и не нужны. Можно прикрутить конечно, но это скорее для красоты.
knuebok 30/11/15 Пнд 22:30:07 #163 №315095 
>>315044
>>315060
Если интересно, то вот как это реализовано в metamath proof explorer: есть множество "модель вещественных чисел, реализованная в виде сечений Дедекинда", - http://us.metamath.org/mpegif/df-nr.html - это "временное множество" единственная цель которого - определить C как квадрат этого множества с некоторой структурой : http://us.metamath.org/mpegif/df-c.html после чего, R определяется как "модель сечений дедекинда" x {0} - http://us.metamath.org/mpegif/df-r.html и доказывается абсолютно строго это самое включение: http://us.metamath.org/mpegif/axresscn.html

>>315092
Для матана на многообразиях нужны, а другим профессионально заниматься не особо осмысленно, мне кажется.

>>315073
>>315087
Это дело вкуса и эстетство.
Аноним 30/11/15 Пнд 22:39:31 #164 №315103 
>>315055
Ну это же преподаватель матанализа, зачем ты с ним так строго? Ему сочувствовать надо, он же не виноват в том что он преподаватель матанализа?
Аноним 30/11/15 Пнд 22:44:38 #165 №315107 
Аноны, а как вы определяете математика? Есть ли какой-то критерий? К примеру, человек, занимающийся численными методами, математик? Есть ли какой-то минимум, который должны знать абсолютно все математики и без которого нельзя назвать человека математиком? Скажем, рандомный преподаватель матанлиза — математик?
Аноним 30/11/15 Пнд 22:45:34 #166 №315109 
>>315107
Щас какой-то вербитодаун напишет про теорему Атьи-Зингера, инфа сотка.
Аноним 30/11/15 Пнд 22:52:12 #167 №315114 
>>315109
Да знаю я это. В прошлом треде похожий вопрос я же и задавал. Надеюсь, в этот раз будут ещё какие-то мнения. Просто у меня недавно вышел спор на эту тему. Каждый ли прикладной математик ­— математик? Любой ли преподаватель математических дисциплин — математик? Можно ли считать математиком человека, не знающего, к примеру, теории категорий/пучков/топосы/спектры/когомологии/etc?
knuebok 30/11/15 Пнд 22:56:23 #168 №315117 
>>315114
Я напишу что я думаю: мне кажется красота математики как раз и состоит в том, что её "актуарная часть" связана с "фундаментальной частью", при этом связана обоюдно - идёт постоянная двухсторонняя подпитка. И для нормального развития математики нужны как прикладники так и фундаменталисты, поэтому, мне кажется, критерий должен быть максимально общий, вида: "решает математические задачи, формулирует свои результаты по схеме определение-доказательство-теорема - значит математик". Таки дела.
Аноним 30/11/15 Пнд 23:08:56 #169 №315125 
>>315117
>И для нормального развития математики нужны как прикладники так и фундаменталисты
Я думаю мир стал бы прекраснее без картофанного матана.
Аноним 30/11/15 Пнд 23:12:59 #170 №315128 
>>315125
Если бы в мире не было картофанного матана, ты бы щас не за пекой сидел, а в поле пахал на барина.
Аноним 30/11/15 Пнд 23:17:49 #171 №315130 
>>315117
>решает математические задачи, формулирует свои результаты по схеме определение-доказательство-теорема - значит математик
А как узнать, что он что-то доказывает? Как вообще искать работы современных учёных? Как узнать, когда у человека последний раз были какие-то научные результаты?
И что такое в твоём понимании математические задачи? К примеру, улучшение каких-то алгоритмов в численных методах — математическая задача? А моделирование потоков вокруг стержней реактора?
knuebok 30/11/15 Пнд 23:22:37 #172 №315131 
>>315130
>Как вообще искать работы современных учёных? Как узнать, когда у человека последний раз были какие-то научные результаты?
Google scholar, arxiv.
> К примеру, улучшение каких-то алгоритмов в численных методах — математическая задача?
Грубо говоря, всё то, что может быть "сформулировано при помощи аксиом ZFC при достаточном усердии", соответственно, если при улучшении алгоритма доказаны какие-то оценки - то это математический результат, а если это эвристическое размахивание руками вида "проверели на 1000 случаях на данных такой-то компании - вроде работает быстро", - то не математический.
>>315125
Uhh, знал бы ты для решения какой задачи Мочидзука свою теорию строил...
Аноним 30/11/15 Пнд 23:27:15 #173 №315133 
>>315131
> Uhh, знал бы ты для решения какой задачи Мочидзука свою теорию строил..
Я не >>315128, но я думаю, что он ее строил не потому, что хотел доказать abc-гипотезу (хотел, конечно, но это не главное).
" https://youtu.be/hQLxmSgL0yI?t=1355 "
Аноним 30/11/15 Пнд 23:28:34 #174 №315135 
>>315133
блять, кавычки поставил, а все равно привязка ко времени проебалась
22:40
knuebok 30/11/15 Пнд 23:31:37 #175 №315136 
>А моделирование потоков вокруг стержней реактора?
Это комплексный результат. Оценка адекватности модели, то есть насколько модель соответствует реально наблюдаемым процессам - это, по сути, экспериментальная физика, а вот если получена содержательная информация о самой модели математическими методами - то это математический результат.
>>315133
Мне кажется это было основной мотивацией. Я смотрел лекции Михайлова и в них он как раз говорил о том, что очень часто мощные фундаментальные теории строились вокруг решения конкретных задач, и даже если они задачу не решали, они оказывались интересны сами по себе. Это именно то, что я хотел сказать.
Если когда-нибудь эта связь будет утрачена (фундаментальные математики будут "свысока" смотреть на конкретные задачи), то новые фундаментальные теории не будут столь глубокими. Если вообще будут.
Аноним 30/11/15 Пнд 23:38:42 #176 №315138 
>>315107
Все вопросы, является ли кто-то X-истом решаются сообществом X-истов. Причём, вес имеет не только мировое, но и любое более-менее крупное сообщество, например региональное: российское или американское. И в российском, и в американском математических сообществах, человека, занимающегося численными методами пока признают математиком, а в сообществе читателей вербитятника, например, - нет. Но население его бложика само по себе три с половиной рашкофобских школьника, так что его мнение вряд ли можно считать весомым.
knuebok 30/11/15 Пнд 23:41:57 #177 №315139 
>>315138
Мне кажется его вопрос был о каком-то "вселенском, абстрактном алгоритме" распознавания математической работы, а не о практических аспектах. Так-то понятно, что если большинство математиков называют результат математическим, то наверное так-то оно и есть. Но бывают и спорные случаи, например, что делать с теоремами, которые доказаны пруверами? Гротендик не признавал доказательство теоремы о четырёх красках, например.
Аноним 30/11/15 Пнд 23:42:39 #178 №315140 
Посоветуйте чего почитать по истории математики, из вышедшего недавно. Интересуют обзоры, или по крайней мере сведения о всяких умерших теорий XIX и начала XX века.
Аноним 30/11/15 Пнд 23:48:02 #179 №315141 
>>315135
Ничего не проебалось.
Аноним 01/12/15 Втр 00:03:57 #180 №315145 
>>315131
>Google scholar, arxiv.
Спасибо, учту. А всякие индексы там можно посмотреть, а то я не нашёл?
Ладно, в этом направлении я понял твою точку зрения. А что скажешь насчёт обязательного минимума для математика?

>>315138
>а в сообществе читателей вербитятника, например, - нет
Не скажу, что отношусь к ним. Если честно, весь этот спор вышел, из-за того что мне начали рассказывать, какой %преподаватель-нейм% крутой математик, на что у меня возникло странное чувство… в общем, негодование. Вот я и захотел узнать, можно ли считать рандомного преподавателя математиком, если он занимается чем-то прикладным. Во всяком случае я не могу поставить таких людей в один ряд с людьми, занимающимися, так скажем, теоретической математикой. Совершенно разные уровни.

>>315139
В общем, да, примерно так.
Аноним 01/12/15 Втр 00:14:42 #181 №315147 
>>314306 (OP)
Аноним 01/12/15 Втр 00:17:56 #182 №315148 
14489182764860.png
>>315145
>Спасибо, учту. А всякие индексы там можно посмотреть, а то я не нашёл?
Пикрил.

>Во всяком случае я не могу поставить таких людей в один ряд с людьми, занимающимися, так скажем, теоретической математикой. Совершенно разные уровни.
Ну, ты это зря. Вот Гротендик писал в своих "Урожаях и Посевах", - что гонор и высокомерие - это очень плохо и недостойно. И рассказывал случай тоже про то, как какой-то чувак просил его рецензию, а он даже смотреть не стал, аргументировав тем, что там картофан какой-то и он всё это уже видел. И вот он некоторую часть книги убивается по-поводу того, как это было неправильно. А вообще есть вот Тао, который теорвером, теорией чисел и комбинаторикой в последнее время занимается, филдсовский лауреат, на секундочку. У него тоже "не тот же уровень", что и у людей, которые занимаются пучками и высшими топосами?
knuebok 01/12/15 Втр 00:26:11 #183 №315150 
Забыл ответить.

>А что скажешь насчёт обязательного минимума для математика?
Мне кажется, что в современных реалиях, недостижимый идеал - это если после обучения ты можешь ответить на любой вопрос из http://www.math.harvard.edu/graduate/quals/topics/ без предварительной подготовки. Достижимый идеал - это новый куррикулум Вербицкого (который вполне адекватен) - http://verbit.ru/Job/HSE/Curriculum/all.txt
Аноним 01/12/15 Втр 00:38:23 #184 №315152 
>>315148
>что гонор и высокомерие
Да нет, ты не подумай, у меня нет ни гонора, ни высокомерия, ибо себя я тем более не могу назвать математиком. Может быть, никогда им и не стану. Тут вопрос в другом скорее. Я бы сказал, вопрос в авторитете. Когда смотришь на людей, которые реально занимаются математикой, получают какие-то результаты, и на людей, которые преподают, но об их заслугах ты ничего не знаешь, непроизвольно проводишь деление. Это всё субъективно, но у меня вообще исчезает всякое уважение к преподавателю, если он не стремится проявить себя как учёный. И как таких преподавателей назвать математиками? Наверно, я не прав. Но апеллировать к своим внутренним ощущениям я тоже не могу. Вот я и пытаюсь рационально объяснить свои чувства.

>>315150
Интересно, посмотрю.
Аноним 01/12/15 Втр 00:39:40 #185 №315153 
Что такое тензоры нулевого, первого, второго, высших рангов?

Где на такой вопрос найти ответ?
Аноним 01/12/15 Втр 00:57:09 #186 №315155 
>>315140
Морис Клайн
Аноним 01/12/15 Втр 01:17:51 #187 №315162 
>>315128
Я готов страдать ради красоты
Аноним 01/12/15 Втр 01:31:50 #188 №315165 
>>315155
>Морис Клайн
Утрата определённости или что? Там в общих чертах же. Интересно, может кто-то с подробностями разбирал конкретные теории прошлого.
Аноним 01/12/15 Втр 01:45:13 #189 №315166 
>>315152
> Это всё субъективно, но у меня вообще исчезает всякое уважение к преподавателю, если он не стремится проявить себя как учёный.
А если, допустим, человек понимает, что не может работать наравне с топовыми специалистами (а 99.9% людей не могут), но не имитирует научную деятельность, публикуя работы по каким-то малоактуальным и не особо интересным проблемам (типа "метод решения вот этого дифура, который возник, когда я моделировал явление, когда мы с женой шли по берегу и было туманно"), а делает что-то реально полезное? Например организация всяких конкурсов и олимпиад среди школьников мне кажется очень важным делом. Даже отвечать на вопросы на math.stackexchange - важное дело.
Аноним 01/12/15 Втр 01:46:36 #190 №315167 
>>315138
> Но население его бложика само по себе три с половиной рашкофобских школьника
Сходу могу назвать человек пять-десять серьезных математиков, которые сидят там. И неисчислимое количество студентов/аспирантов/молодых постдоков.
Аноним 01/12/15 Втр 03:20:54 #191 №315180 
>>315166
>делает что-то реально полезное?
Значит, это хороший преподаватель. Такие у меня вызывают уважение. Таких мало, ибо я в основном вижу лишь высокомерие со стороны преподавателей, словно они пытаются студентам доказать, как много они знают и какие мы тупые. Оно, конечно, так, но зачем?
knuebok 01/12/15 Втр 03:48:39 #192 №315183 
>>315166
>ут), но не имитирует научную деятельность, публикуя работы по каким-то малоактуальным и не особо интересным проблемам (типа "метод решения вот этого дифура, который возник, когда я моделировал явление, когда мы с женой шли по берегу и было туманно"),

Актуальность - это по сути характеристика того, насколько твоя работа потенциально может помочь всяким топовым математикам на вершине социальной пирамиды, - то есть характеристика чисто социальная; а интересно, мне кажется, может быть и с методами решения дифуров при моделировании каких-то там процессов (в конце концов, в чём фундаментальное отличие дифуров, моделирующих явления при туманной погоде, от Навье-Стокса, моделирующего какие-то там потоки в жидкостях?). Я считаю, что для нормального функционирования математического здания нужны и те, кто на подсосе у Великих (филдсовских лауреатов, IHES, макс планк) и занимаются актуальной и сверхсовременной(тм) математикой, и те, кто задрачиваются всю жизнь по проблеме Борсука, и какие-нибудь шизики, чья жизненная мечта построить "правильную теорию множеств", при том эта "правильность" дефенируется лишь их внутренним разумением и интуицией.

>>315180
Это плохие преподаватели и плохие люди. А бывают и хорошие.
Аноним 01/12/15 Втр 04:17:36 #193 №315184 
>>315183
>занимаются актуальной и сверхсовременной(тм) математикой, и те, кто задрачиваются всю жизнь по проблеме Борсука, и какие-нибудь шизики, чья жизненная мечта построить "правильную теорию множеств"
Они абсолютно равноценны.
Аноним 01/12/15 Втр 08:42:47 #194 №315194 
>>315092
>Для матанализа они и не нужны
Немного странная позиция. Зачем что-то оценивать по нужности для матанализа? Матанализ не самоценен, вовсе нет. Напротив, это необходимость изучать матанализ должна быть обоснована. Студент учится, чтобы стать учёным - читать и писать научные статьи. Зачем студенту нужен матанализ?
Аноним 01/12/15 Втр 10:16:19 #195 №315198 
>>315167
Назови. И в каком смысле "сидит"?
Аноним 01/12/15 Втр 10:16:40 #196 №315199 
>>315107

Как минимум настоящий математик должен уметь корректно определять N. Остальное дело вкуса.
Аноним 01/12/15 Втр 10:18:47 #197 №315200 
>>315183
А что сейчас актуально и современно?
Аноним 01/12/15 Втр 10:24:16 #198 №315201 
>>315183

> что для нормального функционирования математического здания нужны и те, кто на подсосе у Великих

Ну и хуйли с того? Для нормального функционирования еще и мусорщики нужны и чистильщики сортиров.
Аноним 01/12/15 Втр 10:24:34 #199 №315202 
>>315200

Найти корректное определение N.
Аноним 01/12/15 Втр 11:36:42 #200 №315214 
Вот вам задача на графы.

Есть граф, в котором вершины раскрашены в какие-то цвета. Каждому цвету c поставим в соответствие целое t[c]. Перекрашивание вершины v из c1 в c2 происходит следующим образом:
1) Если t[c1] = 0, такая перекраска невозможна.
2) Если t[c1] = 1 и после перекраски будет существовать ребро, у которого оба конца имеют цвет c1, такая перекраска невозможна.
2) v перекрашивается из c1 в c2.
3) t[c1] уменьшается на 1.
4) t[c2] увеличивается на 1.

Есть простой (без петель и кратных ребер) неориентированный граф, все вершины которого покрашены в 1, t[1] = 2, t[2] = 0. Нужно научиться за полиномиальное от количества вершин в графе время определять, можно ли перекрасить все вершины в цвет 2 по описанным выше правилам.

Собственно, эта задача - обобщение задачи о волке, козе и капусте, которых надо переправить на другой берег так, чтобы никто никого не съел. Только животных здесь много и перевозчика 2; один их них может перевозить, а другой следить за животными на берегу.
Аноним 01/12/15 Втр 12:31:23 #201 №315221 
>>315214
Графьям не гоже превозкой скота и снеди заниматся. Пшел отсюда, смерд.
Аноним 01/12/15 Втр 13:09:20 #202 №315230 
>>315221
Я нормально сформулировал задачу. Последний абзац - это не условие. Так что не выебывайся.
Аноним 01/12/15 Втр 14:39:08 #203 №315254 
>>315214
>математика
>задача на графы
>обобщение задачи о волке, козе и капусте
О великие силы.
Аноним 01/12/15 Втр 14:39:45 #204 №315255 
>>315221

Но ведь граф это исходное понятие, у маклейна через него категории определяются, вы их любите.
Аноним 01/12/15 Втр 15:14:52 #205 №315261 
Посоны, как вы относитесь к нестандартному анализу? Какие книжки-учебники можно почитать для введения?
Аноним 01/12/15 Втр 15:29:42 #206 №315263 
>>315214
Формулировка в начале непонятна вообще. Почему нельзя три раза подряд перевезти с первого на второй бере, если ничего плохого не происходит? Где там второй перевозчик?
Задача же про перевозчика тривиальна, так как почти всегда нельзя. Граф вражды должен быть почти пустым, так как на береге без перевозчика он должен быть пустым, а за раз перевозим не более двух. Так что с одним перевозчиком получается граф, в котором все ребра соединены с одной из двух фиксированных вершин. Если два перевозчика (один из которых может только следить), то видимо что-то в стиле двудольного плюс две вершины.
Аноним 01/12/15 Втр 15:58:09 #207 №315267 
>>315165
Ну хуй знает, попробуй Френкеля, Основания теории множеств. Так есть разборы старых теорий.
Аноним 01/12/15 Втр 16:50:29 #208 №315272 
14489778295830.png
Товарищи, помогите. Что значит ex. в пикрелейтеде? Исключения?
Аноним 01/12/15 Втр 17:13:49 #209 №315275 
>>315272
exercise
Аноним 01/12/15 Втр 17:14:22 #210 №315276 
>>314334
Прости, я может быть туплю, но я не понимаю, как можно доказать теорему Банаха-Тарского без аксиомы выбора. Можешь простыми словами разъяснить?
Аноним 01/12/15 Втр 17:20:18 #211 №315277 
Есть ли в топологии такое понятие, как алгебра? И что оно означает?
Аноним 01/12/15 Втр 17:20:47 #212 №315279 
>>315277
Есть ли у множеств в топологии такое понятие, как алгебра
самофикс
Аноним 01/12/15 Втр 17:37:40 #213 №315282 
>>315263
> то видимо что-то в стиле двудольного плюс две вершины.
Да, это ответ. Ну не совсем ответ конечно. По идее надо доказать, что граф двудольный без двух вершин <=> граф можно перекрасить таким образом. Но вообще это правильно.
Аноним 01/12/15 Втр 17:37:57 #214 №315283 
>>315272
Example.
Аноним 01/12/15 Втр 17:50:35 #215 №315287 
>>315214
Я чет проебался. Забыл, что помимо перекраски можно для любого с1 такого, что t[c1] >= 1 и любого с2 сделать следующее:
1) уменьшить t[c1] на 1;
2) увеличить t[c2] на 1.

Просто хотел нормально задачу сформулировать, а не в терминах козы и капусты, но чет не очень получилось.
Аноним 01/12/15 Втр 18:07:15 #216 №315289 
>>314306 (OP)
Если в аксиоматике теории множеств (скажем, в ZFC) аксиому пустого множества заменить на противоположную ей (любому множеству принадлежит хотя бы один элемент), будет ли такая аксиоматика непротиворечива? Какие примечательные утверждения, аналогичные утверждениям теории множеств, будут в ней выводимы, а какие - нет?
Аноним 01/12/15 Втр 18:37:21 #217 №315294 
14489842418120.png
Кто подскажет как найти решение к задаче?
Аноним 01/12/15 Втр 18:39:35 #218 №315295 
>>315083
Любое построение вещественных чисел должно быть эквивалентно остальным построениям (в которых 0,(9) вполне существует. То, что 0,(9) не строится по Зоричу я не увидел, может тупой сильно.
Аноним 01/12/15 Втр 19:47:13 #219 №315309 
14489884338840.png
Надо тип дать определение функции по Коши и по Гейне или чет типа того. Но я хуй знает, что делать с бесконечностью, куда ее сувать вообще. Я вообще правильно начал?
Аноним 01/12/15 Втр 20:00:08 #220 №315310 
14489892082560.png
>>315309
Что означает algebra в топологии? То и означает, алгебру?
Аноним 01/12/15 Втр 20:22:17 #221 №315317 
>>315309
Немножечко Коши
Для сердца и души
Аноним 01/12/15 Втр 20:29:51 #222 №315319 
>>315289
>будет ли такая аксиоматика непротиворечива?
Нет.
Пусть M - множество.
Положим P(x) как x != x и воспользуемся схемой выделения, применив P(x) к M. Получим пустое множество, что противоречит аксиоме.

Кстати, аксиома пустого множества вытекает из аксиомы существования бесконечного множества и схемы выделения.
Аноним 01/12/15 Втр 20:50:06 #223 №315323 
>>315319
А что будет, если убрать схему выделения? Потому что это выглядит как чит в данном случае.
Аноним 01/12/15 Втр 21:10:40 #224 №315328 
>>315323
Умрёт понятие "подмножество".
Аноним 01/12/15 Втр 21:15:23 #225 №315329 
>>315255
Да здесь же просто дрочеры википедии сидят, а не математики. Кстати, кого сегодня обоссываем, господа?
Аноним 01/12/15 Втр 21:18:31 #226 №315331 
>>315329
Историков.
Аноним 01/12/15 Втр 21:21:56 #227 №315333 
>>315261
> к нестандартному анализу
Без задач.
Успенского "Что такое нестандартный анализ" читни. Ну или там "Прикладной нестандартный анализ" Дэвиса.
Аноним 01/12/15 Втр 21:24:01 #228 №315335 
>>315309
Определение предела, ты имел в виду, наверное.

Бесконечность особый случай. Предел в точке a равен + бесконечности, если для любого M > 0 найдется такая окрестность а, что для всех x в этой окрестности f(x) > M. В случае - бесконечности предел определяется аналогично.
Аноним 01/12/15 Втр 22:36:40 #229 №315340 
Аноны, что можно по криптографии почитать?
Аноним 02/12/15 Срд 05:11:02 #230 №315372 
Поясните за Рябушко. Норм для освоения матана?
Аноним 02/12/15 Срд 05:52:33 #231 №315375 
>>315340
Книги
Аноним 02/12/15 Срд 09:15:18 #232 №315378 
>>315333

Успенский считает, что нельзя дать определение N.
Аноним 02/12/15 Срд 09:30:03 #233 №315381 
>>315378
Я его не читал, но я тоже так считаю. Чтобы читать и различать символы между собой, нам нужны конечные строки. Работая с конечными строками, мы используем натуральные числа.
Аноним 02/12/15 Срд 10:23:45 #234 №315389 
>>315372
Ноунейм не может сделать годноту.
Аноним 02/12/15 Срд 10:24:32 #235 №315390 
>>315389

Биг Босс смог.
Аноним 02/12/15 Срд 10:25:20 #236 №315391 
>>315390
Ты про лекции фрика Босса, в которых упоминается Господь Боженька?
Аноним 02/12/15 Срд 10:34:13 #237 №315392 
14490416534070.png
>>315391
Аноним 02/12/15 Срд 10:37:10 #238 №315393 
>>315392
>>315391
Всё нормально написал, чего это тебя бомбит-то так.
Аноним 02/12/15 Срд 10:43:11 #239 №315395 
>>315393
В самом деле, чего это я.
Крупному рогатому скоту потребен специальный корм.
Аноним 02/12/15 Срд 11:02:47 #240 №315399 
>>315381

Толково.
Аноним 02/12/15 Срд 11:05:05 #241 №315400 
>>315395
Илитизм в исполнении несведующего школьника в такой достопочтенной тематике как /sci/ выглядит уморительно и нисколько не обидно. Таким эффективнее в би заниматься.
Аноним 02/12/15 Срд 11:07:27 #242 №315401 
Привет, математикам.
Помогите решить пример, пожалуйста. В интернете ничего не нашел:
Линейный оператор f: V3->V3 в базисе (i,j,k) имеет матрицу А.
Описать геометрический смысл оператора, если задана А.
В данном случае A - единичная матрица.
Аноним 02/12/15 Срд 11:16:37 #243 №315402 
>>315401
Оператор оставляет все точки пространства на месте. Это тождественное преобразование пространства, ни одна точка не двигается.
Аноним 02/12/15 Срд 11:28:50 #244 №315405 
>>315402
Спасибо!
Я правильно понимаю, что тогда оператор с матрицой
(1)(0) (0)
(0)(-1)(0)
(0)(0) (-1)
инвертирует j и k, оставляя i на месте?
Аноним 02/12/15 Срд 11:40:23 #245 №315409 
>>315405
Умножь этот оператор на столбец из трех координат и посмотри сам
Аноним 02/12/15 Срд 11:46:47 #246 №315410 
>>315409
Еще раз спасибо, все понятно.
Аноним 02/12/15 Срд 15:31:39 #247 №315474 
А давайте еще подумаем над задачей из предыдущего треда:
можно ли разделить плоскость с двумя выколотыми точками на непересекающиеся окружности? Ни одна окружность не может проходить через выколотую точку.
Аноним 02/12/15 Срд 15:38:18 #248 №315477 
>>315474
Можно.
ЯСКОЗАЛ!!
Аноним 02/12/15 Срд 15:42:04 #249 №315479 
>>315333
Чому без задач? Разве такой подход не упрощает многое?
Аноним 02/12/15 Срд 15:46:42 #250 №315481 
>>315276
А разве её можно без AC доказать?
Аноним 02/12/15 Срд 15:48:38 #251 №315482 
>>315481
Нет. Без аксиомы выбора нельзя даже доказать, что любое бесконечное множество содержит счётное подмножество.
Аноним 02/12/15 Срд 15:49:35 #252 №315483 
>>315378
>>315381
>>315399
Прекрати семёнить, N-петух. Все тебя уже обоссали и забыли. Иди в свой тред, там и общайся сам с собой.
Аноним 02/12/15 Срд 15:58:29 #253 №315487 
>>315483

Кнуёбку НЕПРИЯТНО что его опустили в споре??? DEAL WITH IT, машка.
Аноним 02/12/15 Срд 15:59:01 #254 №315488 
14490611416480.jpg
>>315487
Аноним 02/12/15 Срд 16:01:56 #255 №315489 
>>315487
Дрочишь на Пеано небось по ночам?
Аноним 02/12/15 Срд 17:10:56 #256 №315502 
Допустим я сейчас стою около казино/имею какой то казиносим на телефоне.
Стоит ли ставить все деньги или по одной копейке, если количество денег прямо влияет на шанс выиграша? Например, одна копейка дает шанс выигрыша половину процента, две копейки процент, двадцать копеек десять процентов и так до сотки.
Сразу скажу что до сотки доводить шансы не выгодно, выигрыш маленький.
Аноним 02/12/15 Срд 17:13:30 #257 №315503 
>>315502
Без размера выигрыша дискуссия не имеет смысла. Сформулируй задачу в математических терминах.
Аноним 02/12/15 Срд 17:13:50 #258 №315504 
>>315489

Зачем, если я не считаю его аксиомы определением?
Аноним 02/12/15 Срд 17:14:43 #259 №315505 
>>315503
Допустим выигрыш две копейки, с шансом на сто копеек.
Я вообще мимо проходил и решил спросить, так что не могу в термины.
Аноним 02/12/15 Срд 17:14:47 #260 №315506 
>>315504
Очевидно затем что он смог определить N, а ты до сих пор не можешь.
Аноним 02/12/15 Срд 17:16:59 #261 №315508 
>>315505
>так что не могу в термины.
Здесь не телепаты сидят, поэтому если хочешь получить ответ - придётся учиться.
Допустим ты ставишь ставку A.
Каков будет выигрышь и какова будет вероятность выигрыша если выражать его/её через A?
Аноним 02/12/15 Срд 18:02:14 #262 №315515 
>>315389
Вообще я сейчас полистал второй том - задачки довольно интересные.
Аноним 02/12/15 Срд 18:08:10 #263 №315517 
>>315506

Он тоже не считал свои аксиомы определением.
Аноним 02/12/15 Срд 18:43:00 #264 №315528 
>>315517
Ишь ты, как распетушился.
Давно на холодец тебя не пускали?
Аноним 02/12/15 Срд 19:39:37 #265 №315550 
>>315335
Здорово у етбя получается. А когда -бесконечность, f(x) разве не должна быть меньше M?
Аноним 02/12/15 Срд 19:44:44 #266 №315551 
>>315550
>разве не должна быть меньше
Ну тип поясни пожалуйста за это.
Аноним 02/12/15 Срд 20:05:40 #267 №315556 
>>314306 (OP)
Есть что нибудь по мат физике для уравнения теплопроводности, где для дубоватых ребят разжаловано все что то можно и нельзя.
С аналитическим решением и численым явным.
Аноним 02/12/15 Срд 20:21:13 #268 №315559 
>>315550
Ну я же написал что в минус бесконечности определяется аналогично, то есть тебе нужно просто немного подправить определение. Очевидно, что да, должна быть меньше.
Аноним 02/12/15 Срд 20:36:09 #269 №315560 
>>315559
> в минус бесконечности
фикс:
в случае, когда предел равен минус бесконечности
Аноним 02/12/15 Срд 20:40:11 #270 №315562 
>>315391
>>315392
Поссал на дауна-физика.
Аноним 02/12/15 Срд 20:41:22 #271 №315563 
Везде говорится, что идеал кольца называется простым, если фактор-кольцо по нему является полем. А если я не хочу строить фактор-кольцо, как определить простоту идеала?
Аноним 02/12/15 Срд 21:17:35 #272 №315566 
>>315563
Пусть R - кольцо, I - идеал, a и b - элементы R.
I прост <=> если ab принадлежит I, то a принадлежит I или b принадлежит I.
Аноним 02/12/15 Срд 21:18:02 #273 №315567 
>>315566
Причём I не равно R.
Аноним 02/12/15 Срд 21:21:33 #274 №315568 
>>315566
>если ab принадлежит I, то a принадлежит I или b принадлежит I
Как из этого условия следует, что факторкольцо по такому идеалу не имеет делителей нуля? Может ли в каких-то случаях оказаться, что произведение элементов a и b, не принадлежащих идеалу, принадлежит идеалу? Как это объяснить?
Аноним 02/12/15 Срд 21:42:56 #275 №315569 
>>315563
>>315568
Фактор кольцо по простому идеалу не поле, это область целостности.
Поле это фактор по максимальному идеалу. Любое поле - область целостности, поэтому любой максимальный идеал - простой, но не наоборот.
Вообще, все эти фактормемчики правильно себе представлять так: если ты факторизуешь по идеалу/нормальной подгруппе/подмодулю/etc, заданному каким-то выражениями, то ты по сути строишь новый объект, где у тебя есть дополнительные соотношения между элементами, а именно, эти выражения, участвующие в идеалах - ноль (ну или единица, если мы говорим о группах - нейтральный элемент короч).
Поэтому простота идеала "если ab принадлежит I, то a принадлежит I или b принадлежит I" означает "если выражение ab равно нулю, то либо a = 0 либо b = 0" что и означает по определению, что делителей нуля нет. Конечно, это всё доказывается строго с нудным расставление чёрточек над буквами, но суть в этом.
Аноним 02/12/15 Срд 22:05:36 #276 №315571 
>>315477
Предъяви разбиение.
Аноним 02/12/15 Срд 22:06:42 #277 №315572 
>>315571
Множество всех окружностей нулевого радиуса с центрами в каждой точке покрываемого пространства.
Аноним 02/12/15 Срд 22:20:35 #278 №315574 
>>315572
Не думал, что есть кто-то настолько тупой, что он не догадается, что точка не считается окружностью в этой задаче, и что надо это явно прописывать.
Аноним 02/12/15 Срд 22:29:09 #279 №315576 
>>315528

Я твою мамку-шлюху могу на холодец пустить. Проблемы???
Аноним 02/12/15 Срд 22:34:29 #280 №315579 
>>315568
Пусть I - такой идеал в кольце R. Нуль кольца R обозначим как 0.
Введём отношение эквивалентности ~ следующим образом.
Пусть a и b - элементы R. a~b, если a-b ∈ I.

Это действительно отношение эквивалентности.
Поэтому оно разбивает R на классы эквивалентности.

Далее докажем, что класс эквивалентности нуля равен I.
Известно, что 0 ∈ I, так как I - идеал.
Пусть x ∈ I. Тогда x = x-0, то есть x-0 ∈ I, то есть x~0.
Пусть x~0. Тогда x-0 ∈ I, то есть x ∈ I.

Напомню некоторые свойства факторкольца по идеалу.
Факторкольцо состоит из классов эквивалентности.
Класс эквивалентности элемента x обозначим как [x].
Будем говорить, что x является представителем класса [x].
[a] = тогда и только тогда, когда a~b.
Операции в факторкольце сводятся к операциям в R над представителями классов.
Нулём факторкольца служит класс эквивалентности нуля [0].
По ранее доказанному, I = [0].

Докажем, что в факторкольце по I нет делителей нуля.
Предположим, [a] = [0].
По определению умножения, [a] = [ab], то есть [ab] = [0].
По определению равенства классов, ab~0.
Так как [0] = I, ab ∈ I.
Значит, a ∈ I или b ∈ I, то есть [a] = [0] или = [0].
Что и требовалось доказать.
Аноним 02/12/15 Срд 22:45:49 #281 №315583 
>>315579
>или = [0].
мелкофикс
Аноним 02/12/15 Срд 22:50:02 #282 №315584 
>>315583
>или [ b ] = 0
мелкофикс мелкофикса
Аноним 02/12/15 Срд 22:58:18 #283 №315586 
>>315569
>>315579
Охуенно объяснили, спасибо.
Аноним 02/12/15 Срд 23:31:47 #284 №315591 
>>315574
Ну в следующий раз будешь умнее и будешь свои задачки лучше формулировать.
Аноним 02/12/15 Срд 23:46:41 #285 №315594 
>>315591
Нет, не буду. Если ты даже условие не в состоянии понять, то как ты задачу решишь?
Аноним 02/12/15 Срд 23:51:50 #286 №315595 
>>315594
Гильберта на тебя нету, интуицист проклятый.
Аноним 03/12/15 Чтв 07:15:17 #287 №315602 
Помогите доказать, что любую алгебру с единицей можно задать образующими и соотношениями.

Ну или хотя бы дайте определение алгебры, заданной образующими и соотношениями.
Аноним 03/12/15 Чтв 07:41:45 #288 №315603 
14491177059900.png
>>315602
Аноним 03/12/15 Чтв 10:15:49 #289 №315625 
Задачка из соседнего треда.

Пусть M - множество конечных последовательностей десятичных цифр. Например, 001 ∈ M, 99231 ∈ M.
Назовём бесконечную последовательность десятичных цифр w последовательностью Борхеса, если для любого m∈M верно, что m является подпоследовательностью w.
Существует ли хоть одна последовательность Борхеса?
Аноним 03/12/15 Чтв 10:34:14 #290 №315631 
>>315625
Всякий элемент M можно считать числом в одиннадцатеричной системе счисления (не в десятеричной, чтобы сохранить ведущие нули). Поэтому M счётно.

Для всякого элемента m∈M определено натуральное число l(m) - его длина.

Пусть f - биекция между ℕ и M.
Построим вспомогательную последовательность g(n) таким образом.
g(0) = 0,
g(n) = g(n-1) + l(f(n)), если n>0.

Теперь легко построить последовательность h такую, что подпоследовательность f(i) стоит в ней на месте, начинающемся номером g(i).

Последовательность h и будет нужной нам вавилонской библиотекой.
Аноним 03/12/15 Чтв 14:50:43 #291 №315658 
>>315625
Тащемта никаких особых секретов тут и нет. Просто берешь и без задней мысли строишь последовательность. Может ты имел в виду, что нужно явным образом построить последовательность, то есть иметь возможность вычислить, какая цифра у нее стоит на n-й позиции?
Аноним 03/12/15 Чтв 14:57:24 #292 №315660 
>>315625
>>315658
Если нужно явно, то вот самый простой способ построить w: сначала идут последовательности длины 1, потом длины 2 и т. д. Так как для любого n количество последовательностей длины не больше n + 1 конечно, любая конечная последовательность встретится в w.
Аноним 03/12/15 Чтв 16:45:05 #293 №315666 
А почему аксиомы Пеано не определяют N?
мимо ньюфаг
Аноним 03/12/15 Чтв 17:04:12 #294 №315667 
>>315602
Можно взять пространство образующих, равную самой алгебре. Кстати, если эта алгебра конечна, то порожденная ей тензорная алгебра тоже конечна, так как ее базис состоит из всех возможных мономов исходного базиса. Но что делать с пространством соотношений?

Пару дней уже не могу решить.
Аноним 03/12/15 Чтв 17:20:22 #295 №315670 
>>315666
N-петух, перестань семёнить.
Аноним 03/12/15 Чтв 17:43:46 #296 №315672 
14491538265480.jpg
>>315666
Аноним 03/12/15 Чтв 18:40:01 #297 №315683 
>>315666
Каждый день я хожу с большим черным мешком по земле и собираю натуральные числа. На два полных мешка уходит целый день. Но когда я прихожу домой и сваливаю свое богатство в ванную.... Ммммм....
Аноним 03/12/15 Чтв 19:00:50 #298 №315686 
>Во что переходит вращение треугольника на 120
при всевозможных внутренних автоморфизмах группы
симметрий треугольника?
При автоморфизмах, порожденных отражениями, у меня получился переход во все другие отражения. Но при автоморфизме, порожденном одним поворотом на 120 градусов получился переход в тот же поворот. При автоморфизме, порожденном другим поворотом, получился переход в отражение. Я где-то ошибся, или просто не надо было рассматривать автоморфизмы, порожденные поворотами?
Аноним 03/12/15 Чтв 20:30:06 #299 №315701 
14491638063040.png
14491638063061.png
Как решать первый пик? Я почти уверен что как на втором, а он говорит что нет.
https://www.youtube.com/watch?v=GfUKL30pG08
Аноним 03/12/15 Чтв 20:33:18 #300 №315702 
>>315701
Не путай median и mean.
Аноним 03/12/15 Чтв 20:34:02 #301 №315703 
>>315702
Что? В чём разница?
Аноним 03/12/15 Чтв 20:35:10 #302 №315704 
>>315686
При автоморфизме, порождённом поворотом, поворот должен перейти в поворот. Это же просто последовательность поворотов туда крутишь, сюда. Вообще, каждый поворот по идее и должен в себя переходить.
Аноним 03/12/15 Чтв 20:37:49 #303 №315705 
>>315703
Ну он же объясняет в видео.
Mean - это средневзвешенное, считается как ты показал.
Median - это значение, которое меньше ровно половины всех элементов выборки.
Аноним 03/12/15 Чтв 20:39:12 #304 №315706 
Чем прямая сумма отличается от прямого произведения для векторных пространств?

Можно ли понимать под тензорным произведением факторпространство прямого произведения по некоторому пространству? Какому?

Какие из перечисленных пространств изоморфны между собой V⊕V, V⊗V, V×V?
Аноним 03/12/15 Чтв 20:45:32 #305 №315707 
>>315705
>меньше ровно половины
Тут ответ 26. А он больше 40 равен 50 и меньше 80 элементов выборки. А половина выборки это 85 элементов. Значит такого элемента нет.
Аноним 03/12/15 Чтв 21:14:13 #306 №315712 
>>315707
Если есть повторяющиеся элементы выборки - то просто берешь среддний из отсортированного списка элементов выборки.
Аноним 03/12/15 Чтв 21:15:57 #307 №315713 
>>315712
А ясно. Спасибо. А зачем такие сложности? Для чего это используют?
Аноним 03/12/15 Чтв 21:20:43 #308 №315714 
>>315713
Это используется потому что медиана в отличии от меаны оценка стабильная. Т.е. если у тебя будет выборка из 100 элементов, 99 из них будут иметь значения из множества [-1,0,1] и один элемент будет иметь значение допустим 10000 - то среднее будет сильно смещено, а медиана будет как раз где-то в этом множестве. Таким образом сатисты борятся с выбивающимися значениями (полученными, к примеру, по ошибке, из-за сбоя оборудования или чего еще).
Аноним 03/12/15 Чтв 21:25:38 #309 №315715 
>>315714
Всё понятно. Спасибо.
Аноним 03/12/15 Чтв 21:42:03 #310 №315720 
сап, матаны, у меня вопрос. есть прямая на плоскости, есть уравнение перпендикулярной ей прямой. как сдвинуть первую прямую по второй на какое-то расстояние, например?
Аноним 03/12/15 Чтв 22:16:12 #311 №315724 
А есть задачник для позадрачивать гомологии?
Аноним 03/12/15 Чтв 22:18:00 #312 №315725 
>>315724
Двачую вопрос
Аноним 03/12/15 Чтв 22:34:58 #313 №315728 
>>315724
В книги Прасолова элементы теории гомологий более сотни задач.
Аноним 03/12/15 Чтв 23:52:53 #314 №315734 


Хикканы, росто хочу рассказать вам, какие ваши мамки мрази. Им насрать на ваше внутреннее состояние, им насрать на ваши депрессии, низкий ранг и все что угодно. Вы для них как кастрированные котики - просто забавные зверушки. Их беспокоит только ваше ТЕЛО, т.е. физическая сторона 1) тело сыто 2) тело не болеет. Для них не существует вашей личности. Все что они знают о вашей личности, это что ОН У НАС ДОМАШНИЙ, МАМОЧКУ СЛУШАЕТСЯ ХОРОШИЙ МАЛЬЧИК, ДОБРЫЙ НО ЛЕНИВЫЙ ТОЛЬКО! Поэтому то они не против, что вы хиккуете ежесуточно, ведь на улице ТЕЛО и убить могут или покалечить, а мамка этого не переживет, веди мамка только о себе думает, о своем инстинкте материнском ну и конечно о теле, которое выползло из ее пизды, ОНА ВЕДЬ ТАК МУЧИЛАСЬ КОГДА РОЖАЛА. В тюрьме преступников тоже кормят. Задумайтесь.
Аноним 03/12/15 Чтв 23:54:58 #315 №315735 
>>315734
Накастовал тебе в ротик своих дум.
Аноним 04/12/15 Птн 00:27:17 #316 №315739 
>>315734
Насовал в твою мамашку своего ТЕЛА
Аноним 04/12/15 Птн 01:19:24 #317 №315743 
>>315734
Почему ты выбрал этот тред для вбрасывания?
Аноним 04/12/15 Птн 09:28:18 #318 №315770 
>>315734

Толково.
Аноним 04/12/15 Птн 09:28:52 #319 №315771 
>>315720

Луркай теорему Казухиры Миллера.
Аноним 04/12/15 Птн 10:21:29 #320 №315779 
Комрады, кто английском шарит, помогите перевести следующую подчеркнутую фразу, а то что то я застрял: in completely regular spaces Y the spaces C(Y) and C(Y;c) are, in general, distinct joint topological invariants of Y and E.
Аноним 04/12/15 Птн 10:30:04 #321 №315783 
>>315779
В общем случае в completely regular пространстве Y пространства C(Y) и C(Y; c) - это distinct joint topological invariants of Y and E.

Я не знаю, что такое completely regular и distinct joint topological invariants, но думаю тебе уже должен стать понятен смысл.
Аноним 04/12/15 Птн 10:32:01 #322 №315786 
>>315783
Анон, я знаю, как перевести всю фразу, кроме того, что я подчеркнул: distinct joint topological invariants. Мне понятно каждое слово по отдельности, и даже попарно, но у этого словосочетания есть какой то математический смысл, потому и написал сюда, надеясь на то, что придёт анон-математик и спасёт меня. Весь остальной текст просто для того, чтобы был виден контекст фразы.
Аноним 04/12/15 Птн 10:53:16 #323 №315792 
>>315743
Потому что он - шизик, очевидно же.
Аноним 04/12/15 Птн 11:06:04 #324 №315794 
Поясни за курс математики в хан академии, годно?
Аноним 04/12/15 Птн 11:29:00 #325 №315797 
>>315794
для не математика --- да.
Аноним 04/12/15 Птн 11:48:33 #326 №315802 
>>315792

C точки зрения малолетнего долбоёба - безусловно.
Аноним 04/12/15 Птн 11:58:44 #327 №315803 
>>315794
Есть кайнда общий методологический принцип такой: любой "образовательный" курс в модных нынче на западе интернет-школках от молодых-шутливых-прогрессивных говно на уровне научпопа для 11классников. По крайней мере по математике и более-менее классическим наукам, может своей бизнес-хуите они и нормально учат, не знаю.
Аноним 04/12/15 Птн 12:06:45 #328 №315805 
>>315803
Ну там и курс рассчитан на получение навыков школьного курса.
Аноним 04/12/15 Птн 13:39:07 #329 №315811 
Математику можно изучить только самостоятельно доказывая теоремы и выводя формулы. Все эти видеокурсы - это хуйня для хипстеров.
Аноним 04/12/15 Птн 13:45:28 #330 №315812 
>>315482
Но в оп-посте написано:
" Как побочный эффект, отпадает всякая необходимость в аксиоме выбора, лемме Цорна и им подобных."
knuebok 04/12/15 Птн 14:27:43 #331 №315815 
>>315706
Для конечного произведения суммы двух пространств - ничем. Для бесконечного - отличаютя. Грубо говоря, сумма векторных V_i пространств - это все такие строки того же размера, что и количество слагаемых в сумме такие, что на i-ом пространстве стоит элемент из V_i и на почти всех (кроме конечного числа) местах стоят нули. Произведение - то же самое, но требование, чтобы на почти всех местах стояли нули уже не нужно.
>>315706
>Можно ли понимать под тензорным произведением факторпространство прямого произведения по некоторому пространству? Какому?
>Можно ли понимать под тензорным произведением факторпространство прямого произведения по некоторому пространству? Какому?
Нельзя. Тупо по размерностям не подходит.
>Какие из перечисленных пространств изоморфны между собой V⊕V, V⊗V, V×V?
V⊕V и V×V
Аноним 04/12/15 Птн 14:59:39 #332 №315818 
>>315812
Она отпадает только локально, в конкретно этом разделе общей топологии.
Аноним 04/12/15 Птн 16:57:06 #333 №315828 
>>315811
И к чему ты это кукарекнул, ммм, уёба?
Аноним 04/12/15 Птн 18:37:27 #334 №315844 
14492434474540.png
>>315815
Вроде разобрался.
По поводу второго вопроса: вот, что написано у Миши в листочках по алгебре. Интересное определение. Я так понял, что это из теории представлений и связано с заданием алгебры с помощью образующих и соотношений. Пространство образующих --- прямое произведение. Пространство соотношений --- пространство, порожденное соотношениями линейности.
Кстати, никто не хочет со мной порешать эти листочки до января?
Аноним 04/12/15 Птн 18:43:07 #335 №315846 
>>315844
Ошибся, пространство образующих свободно порождено парами из сомножителей прямого произведения (т.е. суммой произведений конечного числа элементов пары).
knuebok 04/12/15 Птн 19:07:28 #336 №315855 
>>315844
Я не понял, что такое пространство образующих и что такое пространство соотношений, впервые в жизни слышу эти слова в таком порядке.
Векторное пространство, свободно порождённое парами векторов из пространств V,W - это не то же самое, что и произведение векторных пространств. Там значок "х" между V и W понимается не как произведение векторных пространств, а как декартовое произведение множеств (которое никакого отношения не имеет к произведению векторных пространств), а значок < S >, где S - любое множество, понимается как векторное пространство с базисом S.
Миша как всегда сверхаккуратен в формулировках.
Аноним 04/12/15 Птн 19:17:43 #337 №315857 
>>315805
Ну как хочешь. Если хоть капелька обучаемости есть - на таком уровне и на уровне стандартной программы вуза всегда эффективнее читать книжки самому, а вопрошать насчёт непонятного где-то в интернетике.
Аноним 04/12/15 Птн 19:19:33 #338 №315858 
>>315855
Я опять запутался. Чем отличается декартово произведение от прямого произведения для векторных пространств?
Аноним 04/12/15 Птн 19:28:10 #339 №315860 
>>315855
Да верно он вс в общем понял же, не путай нюфага.
Тензорное произведение - это фактор типа декартового точнее свободно-порождённого им векторного пространства по тем соотношениям.
knuebok 04/12/15 Птн 19:29:46 #340 №315861 
>>315858
Декартово произведение AxB - это вообще нечто такое, что не сохраняет структуру векторных пространств, то есть это тупо операция на множествах. Собственно, это просто множество пар. (a,b) где a \in A, b \in B.
Произведение векторных пространств VxW - это множества пар вида (v,w) где v \in V, w \in W. С покоординатными операциями:
(a,b)+(c,d) = (a+c,b+d), lambda(a,b) = (lambda a, lambda b). То есть это векторное пространство тоже.
Если S - любое множество без структуры, то множество комплекснозначных функций на S, принимающих ноль почти во всех, кроме конечного числа, точек - это векторное пространство <S> с базисом S. В терминах Миши - свободно порождённое элементами множества S пространство.

Теперь понятно в чём отличие векторного пространства с базисом V x W (в смысле декартового произведения), или, в терминах Миши, свободно порождённого парами (v,w) \in VxW пространства, от произведения векторных пространств. Например, в произведении векторных пространств есть равенство (a,b)+(c,d)= (a+c,b+d) а в векторном пространстве с базисом V x W (в смысле декартового произведения) - такого равенства нет. (a+c,b+d), (a,b), (c,d) - элементы не связанные никакой структурой. Отфакторизовав это пространство по нужному идеалу мы можем получить тензорное произведение.
knuebok 04/12/15 Птн 19:32:33 #341 №315863 
>>315860
Судя по вопросу
>Я опять запутался. Чем отличается декартово произведение от прямого произведения для векторных пространств?
понял не совсем верно.
Аноним 04/12/15 Птн 19:39:22 #342 №315864 
>>315861
Спасибо.

Аноним 04/12/15 Птн 20:38:58 #343 №315870 
Как научиться бездарю доказывать теоремы?
В жизни ни разу ничего не доказывал. Не понимаю даже принципа как это делается.
Аноним 04/12/15 Птн 21:17:14 #344 №315883 
Почему тут нормальная теорема Стокса http://www.math24.ru/%D1%82%D0%B5%D0%BE%D1%80%D0%B5%D0%BC%D0%B0-%D1%81%D1%82%D0%BE%D0%BA%D1%81%D0%B0.html
а тут какая-то хрень, какие-то многообразия, шо и несет https://ru.wikipedia.org/wiki/%D0%A2%D0%B5%D0%BE%D1%80%D0%B5%D0%BC%D0%B0_%D0%A1%D1%82%D0%BE%D0%BA%D1%81%D0%B0
Аноним 04/12/15 Птн 21:22:44 #345 №315884 
>>315883
Та, что ты считаешь нормальной, рассчитана на студентов технических специальностей, не собирающихся изучать математику, ну или физиков конца позапрошлого века.
Аноним 04/12/15 Птн 21:32:08 #346 №315885 
>>315844
А как же конкретно отфакторизуем? Какое там отношение эквивалентности?
Аноним 04/12/15 Птн 21:35:35 #347 №315886 
>>315870
Чтобы начать ничего особого не нужно, просто интерес и желание докопаться. Мне (не всегда но все же) хватает для решения элементарных задач где надо доказать утверждение.
Попробуй начать с детских книжек. Реймонд Смаллиан (принцесса или тигр итп) например.
Аноним 04/12/15 Птн 21:51:38 #348 №315889 
Господа, а можете пояснить, что именно изучает алгебраическая геометрия? Понятно, что алгебраические многообразия (кривые, поверхности, гиперповерхности). А что именно эта дисциплина пытается узнать о них? Какие-то тонкие инварианты, изучение функциональных полей, ассоциированных с кривой, локализация координатных колец, вообще охуеть. Ради чего это все?
Аноним 04/12/15 Птн 21:53:45 #349 №315890 
>>315886
А как же доказательства "Пускай ... , то ..."?
Как научится то? Если я просто буду перечитывать теорему, то вряд ли мне что-то вдруг на ум придет.
Аноним 04/12/15 Птн 22:28:27 #350 №315893 
>>315890
Начни действительно с книг Саллмана "Как же называется эта книга", "Принцесса или тигр" и такое всё, а там, возможно, самому станет понятно.
Аноним 04/12/15 Птн 22:33:26 #351 №315894 
>>315893
>Саллмана
Смаллина, конечно же, сорян.
Аноним 04/12/15 Птн 22:36:06 #352 №315895 
Смаллиана, блять! Сука, у меня БОЗДОМИР КОНФЬЮЖЕН от этого.
Аноним 04/12/15 Птн 22:52:11 #353 №315900 
>Докажите, что в коммутативной группе всякая подгруппа является нормальной подгруппой.

По определениям получилось, что если a - элемент подгруппы N, то gag^{-1} = gg^{-1}a = a. То есть внутренние автоморфизмы оставляют элементы нормальной подгруппы неподвижными?

У меня в примере получилось, что N - подгруппа вращений треугольника, то автоморфизмы, порожденные вращениями, оставляют неподвижными другие вращения, но автоморфизмы, порожденные отражениями, уже могут переводить одно вращение в другое.

Судя по тому, что формально из a при отображении всегда получается a, этого быть не должно.
Аноним 04/12/15 Птн 23:24:36 #354 №315902 
14492606762840.png
14492606762861.png
>>315889
Читайте, дети, Хартсхорна.
Аноним 04/12/15 Птн 23:32:18 #355 №315904 
>>314829
>Хеллер
>каком-то

Ой, пиздуй отседова.
Аноним 04/12/15 Птн 23:38:53 #356 №315905 
>>315895
> БОЗДОМИР КОНФЬЮЖЕН
??
>>315890
там это и есть
вообще читать примеры доказательств в учебниках тоже конечно помогает
но специально учиться что-то доказывать тупо и ненужно
берешь конкретные несложные задачи и решаешь
Аноним 04/12/15 Птн 23:40:49 #357 №315906 
>>315895

Cмулльян, мудило!
Аноним 04/12/15 Птн 23:49:25 #358 №315908 
>>315902
>Читайте, дети, Хартсхорна
Хм, сегодня в автобусе его читал, а пару дней назад обсуждал с преподавательницей. Изучение только систем уравнений это как-то кисло для дисциплины такого уровня.
Аноним 05/12/15 Суб 00:01:54 #359 №315909 
>>314829
А что у него не так с тян? Всрытый, социопат или тру шизоид?
Аноним 05/12/15 Суб 00:03:05 #360 №315910 
>>315908
>Изучение только систем уравнений это как-то кисло для дисциплины такого уровня.
Какое-то странное утверждение. Если дисциплина "такого уровня" и разрослась из "систем уравнений", то что значит "кисло"? Значит системы уравнений отнюдь не так просты, как тебе кажется.
knuebok 05/12/15 Суб 00:14:08 #361 №315911 
>>315910
Ничего не понимаю в алгебраической геометрии, но из обрывков информации, которые я имею у меня сложилось впечатление, что она гораздо глубже, нежели просто "методы для решения полиномиальных уравнений". Скорее, если мыслить себе геометрию, порождаемую функциями C^{(0)} как "обычную (метрическую, риманову) геометрию", геометрию, порождаемую функциями C^{(\infty)} как "дифференциальную геометрию", множество линейных функций - как "линейную геометрию", то алгебраической геометрии бы соответствовала алгебра полиномов. И правильно смотреть на неё как на изучение "полиномиальных объектов" вообще, точно так же как дифференциальная геометрия изучает "объекты со структурой дифференцирования" а линейная геометрия изучает "линейные объекты".
Аноним 05/12/15 Суб 00:35:20 #362 №315917 
А есть книжечка по алгебрам Ли, но не как стандартный курс по этим самым алгебрам (нильпотентные и разрешимые алгебры Ли, структурная теория полупростых, системы корней и группы вейля, теория представлений над комплексными, ...), а посвящённый собственно рассмотрению алгебр типов A, B, C, D как они устроены непосредственно, где возникают, всякие соотношения-изоморфизмы между ними итп?
Аноним 05/12/15 Суб 12:14:54 #363 №315965 
>>315909

5 из 10, но высокие требования поэтому трахает только элитных шлюх с почасовой оплатой.
Аноним 05/12/15 Суб 12:23:39 #364 №315966 
Поясните мне за дроби.
Почему умножение одной трети на одну треть дает одну девятую, а ту же треть делить на треть дает единицу? Ведь треть от трети это будет как раз одна девятая.
Аноним 05/12/15 Суб 12:31:03 #365 №315969 
>>315966
По определению.
Аноним 05/12/15 Суб 12:43:20 #366 №315970 
>>315969
А если пояснить как для дебила?
Аноним 05/12/15 Суб 13:15:08 #367 №315979 
>>315970
Определение. Произведение дроби a/b и дроби c/d есть дробь ac/bd. Точка.
Аноним 05/12/15 Суб 14:51:00 #368 №315996 
Нихуя не могу в интегралы. Где можно почитать понятные статьи? Матпрофи хуйня, учебники читал.
Аноним 05/12/15 Суб 15:12:20 #369 №315998 
>>315996
В /un/
Аноним 05/12/15 Суб 15:24:19 #370 №316006 
>>315966
Одна треть от 1/3 это 1/3 разделить на 3, няша.
Аноним 05/12/15 Суб 15:26:07 #371 №316008 
Зашкварно ли сидеть на dxdy? Там же все картофанщики.
Аноним 05/12/15 Суб 15:53:31 #372 №316019 
Пусть V - конечномерное векторное пространство.
Если есть некоторый многочлен от линейного оператора на V, т. е. оператор вида
anTn + ... + a2T2 + a1T + a01V,
где 1V - тождественное преобразование V, то его можно разложить на множители как многочлен. Почему так?

В статье на википедии написано, что вместо поля кольцо многочленов можно опредлить над любым коммутативным кольцом. Меня смущает то, что композиция линейных операторов не коммутативна.

Аноним 05/12/15 Суб 16:02:48 #373 №316025 
>>316019
Алсо, я пока что не проходил кольца и все такое, только линейную алгебру. Без этих знаний можно понять ответ на вопрос или стоит временно принять это дело на веру?
Аноним 05/12/15 Суб 16:08:21 #374 №316027 
>>316008
Нет ни одной причины там сидеть, ничего интересного там не происходит. Тут вопросы интересней.
sageАноним 05/12/15 Суб 16:17:41 #375 №316031 
>>315966
когда ты умножаешь 2 на 3, ты берешь двойку 3 раза
когда ты умножаешь 1/3 на себя, ты "берешь 1/3" 1/3 раз, получается 1/9
когда делишь, ты считаешь сколько раз 1/3 уместится в 1/3
примерно так должны в младших классах объяснять
sageАноним 05/12/15 Суб 16:19:35 #376 №316032 
>>316008
помню там кто-то ловера советовал по теоркату, очень хорошая книжка
длинная только больно, функторы только в середине книги
как раз математик был работающий вроде (arxiv читал и все такое)
Аноним 05/12/15 Суб 17:39:46 #377 №316040 
>>316019
a_i это элементы поля или тоже линейные операторы?
Если первое то
> композиция линейных операторов не коммутативна
это конечно так, но композиция оператора самого с собой (и, следовательно, любой свое степени с любой степенью) очень даже коммутативна, это следствие ассоциативности на самом деле. Соответственно кольцо таких полиномов это просто некий фактор обычного кольца полиномов.
Аноним 05/12/15 Суб 19:11:39 #378 №316062 
>>315966
Нас тралят и смеются с нас.
Аноним 05/12/15 Суб 19:36:44 #379 №316066 
>>316032
Я его как-то рекомендовал, потом почитал Гельфанда, Манина - рекомендую его (там нет леммы Ионеды, но про нее можно в вики прочесть).
Аноним 05/12/15 Суб 22:29:33 #380 №316091 
>>316066
А Прасолов, выше упомянутый, как?
Аноним 05/12/15 Суб 22:45:32 #381 №316095 
>>316091
Как земля.
Аноним 06/12/15 Вск 01:46:09 #382 №316106 
>>316095
пальцы вверх кто ненавидит книги прасолова
Аноним 06/12/15 Вск 02:13:40 #383 №316109 
>>316095
>>316106
А, поконкретней, что не так?
И в таком случае, ещё бампану вопрос >>315724
Аноним 06/12/15 Вск 05:45:04 #384 №316121 
Запилите гайд изучения математики (что обязательно прочесть и пр.). От малого к большому, чтобы втянуло.
Аноним 06/12/15 Вск 10:03:16 #385 №316134 
>>316121
http://pastebin.com/y5cjFefW
Аноним 06/12/15 Вск 10:51:40 #386 №316143 
>>316134
Давайте добавим в пасту книги, от которых надо предостеречь ньюфагов. Я начну:
1) Демидович
2) Кнут, Конкретная математика
Эти книги особенно опасны тем, что их искусственная сложность может создать иллюзию, что они нужны, и что если не получается вычислить интеграл от 1 переменной, то чем-то более современным заниматься точно не получится.
Аноним 06/12/15 Вск 11:59:39 #387 №316149 
>>316143
>Эти книги особенно опасны тем, что их искусственная сложность может создать иллюзию, что они нужны, и что если не получается вычислить интеграл от 1 переменной, то чем-то более современным заниматься точно не получится
Двачую этого. Интегралы брал в 9 классе и даже по просьбе решил на 5 контрольную какой-то 40-летней тетке в вечерней школе в далеком 2008. Когда я туда пришел, никто даже ничего не заподозрил и не спросили документы о том, учусь ли я там или нет.
Аноним 06/12/15 Вск 14:15:23 #388 №316203 
>>316143
Ну если кто-то не может понять и научиться применять довольно простой материал, разжеванный везде где только можно, то современная математика действительно не для него.
Аноним 06/12/15 Вск 14:25:20 #389 №316209 
>>316143
>Демидович
Кто-то похуй. Матан и есть матан.
>Кнут, Конкретная математика
Да. Кнут вреден и не нужен.
Аноним 06/12/15 Вск 17:03:57 #390 №316241 
>>316143
Странно, математики гнобят Кнута, а программисты -- превозносят?
Аноним 06/12/15 Вск 17:15:45 #391 №316244 
14494113456370.png
Ваши решения, господа. Доказательство обязательно.
Аноним 06/12/15 Вск 17:22:09 #392 №316247 
>>316244
Зависит от того, сколько раз я мог бы ответитить и от конечного количества возможных вариантов ответа.
Аноним 06/12/15 Вск 17:34:42 #393 №316252 
>>316134
>>316143
Пасты какие-то, шозахуйня, скоро в програмач с их FAQ тред превратите. Насколько вы хорошие математики, кстати, чтобы советовать, что нужно математику, а что нет?
Аноним 06/12/15 Вск 17:34:51 #394 №316253 
>>316241
Я программист и ненавижу Конкретную математику. Задачи там нормальные, но текст читать абсолютно невозможно. Стиль повествования у него просто невыносимый, этакая смесь художественной и технической литературы. К тому же он концентрирует внимание не на природе вещей, а на трюках типа "вот сюда подставим, здесь сделаем вот такую замену, тут домножим".

Аноним 06/12/15 Вск 17:35:32 #395 №316254 
>>316241
Просто программисты - дауны с комплексом быдлокодера, который они сами культивируют в своей среде. Я, например, охуительно понял сортировку merge sort из книги Анания Левитина, а от Кнута проблевался.
Аноним 06/12/15 Вск 17:38:27 #396 №316256 
>>316253
Очень много где в настоящей математике доказательства "трюковые" в той или иной степени. Доказательства в 2.5 строчки или доказательства, получающиеся "шевелением мизинца", зачастую, чистая удача.
Аноним 06/12/15 Вск 18:52:31 #397 №316272 
>>316252
> Насколько вы хорошие математики, кстати, чтобы советовать, что нужно математику, а что нет?
Ну паста не претендует на правильность, это просто мнение.

FAQ, я считаю, надо запилить, потому что есть вопросы, которые повторяются из треда в тред. Вот че я написал: http://pastebin.com/8Sna4WYn
Аноним 06/12/15 Вск 18:58:41 #398 №316275 
>>316244
Скажем, что существует непустое множество S, элементы которого называются осмысленными формулировками.
Скажем, что существует непустое множество A, элементы которого называются возможными ответами.

Вопросом Mes называется следующий набор данных.
1. Непустое конечное множество M, являющееся подмножеством A, элементы которого называются вариантами ответа
Положим, что на M введено отношение линейного порядка.
2. Точка e∈M, называемая правильным ответом
3. Точка s∈S, называемая формулировкой вопроса

Рассмотрим класс Q вопросов.
Рассмотрим функцию ans: Q → A, которая каждому вопросу Mes сопоставляет его правильный ответ e.
Рассмотрим функцию quiz: Q → 2A×S, которая каждому вопросу Mes сопоставляет пару Ms, то есть (варианты ответа, формулировка).
Функцию f: 2A×S → A такую, что f∘quiz = ans, мы будем называть рациональным агентом.
Классы вопросов, для которых существует рациональный агент, будем называть правильно построенными вопросами.

Пусть на картинке написан правильно построенный вопрос.
Тогда один из четырёх вариантов ответа является правильным ответом, и существует рациональный агент, отвечающий на этот вопрос по его формулировке s.

Существование такого рационального агента, однако, противоречит аксиоме фундирования, так как из формулировки следует, что он должен быть своим (наследственным) элементом. Поэтому предположение, что на картинке правильно построенный вопрос, противоречит ZFC. ZFC - единственный инструмент, которым я располагаю. Поэтому у меня нет техники, с помощью которой можно изучать этот рациональный агент.
Аноним 06/12/15 Вск 19:03:37 #399 №316276 
>>316256
>Очень много где в настоящей математике доказательства "трюковые" в той или иной степени. Доказательства в 2.5 строчки или доказательства, получающиеся "шевелением мизинца", зачастую, чистая удача.
Хотя это правда зачастую так, но это недоработки. Если какая-то область понята по-настоящему хорошо, то в ней нет места трюкам - содержательные идеи оказываются отражены в определениях, доказательства же проводятся естественным и ожидаемым образом.
мимо-гротендик
Аноним 06/12/15 Вск 19:05:40 #400 №316278 
>>316275
>ZFC - единственный инструмент, которым я располагаю.
https://en.wikipedia.org/wiki/Aczel's_anti-foundation_axiom
Просвещайся.
Аноним 06/12/15 Вск 19:10:22 #401 №316279 
>>316276
Можно хотя бы одну такую область в качестве примера?
Аноним 06/12/15 Вск 19:11:46 #402 №316280 
>>316279
Ну из наиболее очевидного, теория категорий, общая топология.
Аноним 06/12/15 Вск 19:16:01 #403 №316282 
Кто может помочь мне с задачами по физике?
Аноним 06/12/15 Вск 19:19:06 #404 №316286 
>>316280
>общая топология
Доказательство леммы Урысона нетрюковое? То есть конструкция той непрерывной функции, разделяющей два замкнутых множества - каждый, кто об этом думал, придумал сам?
>теория категорий
Там вообще все доказательства дико трюковые, из-за чего у меня бомбит немерянно. Взять ту же эквивалентность определений adjoint функтора. Или, из простого, доказательство того, что два базиса одного и того же объекта имеют одинаковую мощность - ебался n часов с ним.
Вполне возможно, что просто я тупой, конечно (а я действительно тупой).
Аноним 06/12/15 Вск 19:33:28 #405 №316290 
>>316272
Мне он не нравится. То, что геометрическая прогрессия - это-то конечно правильно, но это куда более мощное средство, и вполне резонно задать вопрос: а почему сумма бесконечной геометрической прогрессии именно такая? Тут, мне кажется, гораздо резоннее пояснить человеку, что 0.(9) и 1.0 - это просто кодирования вещественных чисел, а не сами эти числа. И при построении этого кодирования мы сами же кодируем одно и то же число дробями 0.(9) и 1.0 и истинная причина почему так получается, потому что мы сами же и определили вещественные числа как нечто такое, в чём нету бесконечно малых (аксиома Архимеда) а если бы 0.(9) и 1.0 кодировали разные числа, то 1.0 - 0.(9) нам сразу бы дало это самое "бесконечно малое" число.
То же самое с 1+2+3+... не объяснен смысл и мотивировки этих определений.
Школогайд вроде норм, хотя не знаю, осилил бы я его будучи человеком, ничего не понимающим в математике.
Аноним 06/12/15 Вск 19:51:15 #406 №316294 
>>316286
>Доказательство леммы Урысона нетрюковое?
Да, оно естественным образом восстанавливается из формулировки. По крайней мере я, прямо сейчас, не помня доказательства, за несколько минут его восстановил. Дело в том, что доказательства всех базовых теорем (я разумеется говорил только о них) это просто комбинации нескольких тривиальные манипуляций с данными понятими, основанных непосредственно на определениях.
>каждый, кто об этом думал, придумал сам?
Конечно нет - нужен некоторый уровень математической культуры. Сложно представить ситуацию, когда человек освоил общую топологию, а потом, хотя может вспомнить доказательства большинства классических результатов, не может вспомнить доказательство леммы Урысона.

В теории категорий временами часто возникают несколько тяжеловесные конструкции, но там нигде, из того с чем я знаком (примерно половина Маклейна и, на всякий случай, с общим понятием базиса я не знаком) для доказательств не нужны какие-то оригинальные идеи, всегда достаточно лишь тщательного анализа условия (да, это может требовать времени, я, например, пол дня провозился с леммой Йонеды).
Аноним 06/12/15 Вск 19:58:52 #407 №316297 
>>316290
Невозможно объяснить, чему равна какая-то бесконечная последовательность, не используя понятия предела. А когда ты знаешь определение определа, определить сумму ряда и вывести формулу для геометрической прогрессии очень просто. Формула выводится следующим образом.
Используем школьную формулу для an - bn (можно найти здесь http://math.stackexchange.com/questions/11618/algebraic-identity-an-bn-a-b-sum-limits-k-0n-1-akbn-1-k ).
Дальше
sum q^k от 0 до n = (1 - q^{n + 1}) / (1 - q) (по предыдущей формуле)
Предел этой суммы при n -> \infty равен 1 / (1 - q).

> Тут, мне кажется, гораздо резоннее пояснить человеку, что 0.(9) и 1.0 - это просто кодирования вещественных чисел, а не сами эти числа. И при построении этого кодирования мы сами же кодируем одно и то же число дробями 0.(9) и 1.0 и истинная причина почему так получается, потому что мы сами же и определили вещественные числа как нечто такое, в чём нету бесконечно малых (аксиома Архимеда) а если бы 0.(9) и 1.0 кодировали разные числа, то 1.0 - 0.(9) нам сразу бы дало это самое "бесконечно малое" число.
По-моему, это сильно путано и неконкретно.

> То же самое с 1+2+3+... не объяснен смысл и мотивировки этих определений.
Ну я сам не знаю, какой смысл работать с "суммами" расходящихся рядов. Просто этот вопрос всплывает постоянно и он уже заебал.
Аноним 06/12/15 Вск 20:11:55 #408 №316302 
>>316297
Забыл добавить: то, что нужно посчитать сумму ряда, следует напрямую из определения позиционной системы счисления. А системы счисления в школе проходят на информатике.
Аноним 06/12/15 Вск 22:36:01 #409 №316356 
Если x,y - рациональные числа, то верно ли:
x>y & y<x => x=y
?
Аноним 06/12/15 Вск 22:48:53 #410 №316361 
>>316356
Верно ли что бог есть => я ебал твою мамку?
Аноним 06/12/15 Вск 22:49:33 #411 №316362 
>>316356
Верно.
Аноним 06/12/15 Вск 22:50:39 #412 №316363 
>>316356
алсо x>y <=> y<x, так что твой вопрос это "x>y => x=y", что неверно.
Аноним 06/12/15 Вск 22:52:17 #413 №316364 
>>316361
Да, т.к. ты все равно ебал мою мамку.
Аноним 07/12/15 Пнд 01:45:27 #414 №316382 
Накидайте теорем в стиле "любое разумное пространство изоморфно (в некотором смысле) пространству R^T", ну то есть показывающих, что мол, конечно, множество - это что угодно, но в реальной жизни мы не уходим слишком далеко от действительных чисел.

Алсо, чем учебник Зорича лучше Садовничего-Сендова?
Аноним 07/12/15 Пнд 03:11:34 #415 №316385 
>>316382
https://ru.wikipedia.org/wiki/%D0%A2%D0%B5%D0%BE%D1%80%D0%B5%D0%BC%D0%B0_%D0%A3%D0%B8%D1%82%D0%BD%D0%B8_%D0%BE_%D0%B2%D0%BB%D0%BE%D0%B6%D0%B5%D0%BD%D0%B8%D0%B8
knuebok 07/12/15 Пнд 04:10:26 #416 №316387 
>>316382
Теорема Нёбелинга—Понтрягина: всякое нормальное пространство $X$ со счётной базой размерности $\dim X=n$ гомеоморфно подмножеству пространства $\mathbb R^{2n+1}$.

>Алсо, чем учебник Зорича лучше Садовничего-Сендова?
Второй учебник не смотрел, но из общих соображений: Зорич - не даун, а вот насчёт Садовничего большие сомнения.

Аноним 07/12/15 Пнд 07:44:43 #417 №316392 
>>316382
Побуду капитаном. Любые два векторных пространства одинаковой конечной размерности n над одним и тем же полем изоморфны. В частности, n-мерное пространство над R изоморфно Rn.
Аноним 07/12/15 Пнд 10:51:36 #418 №316413 
>>316382
Есть теорема о существование борелевского изоморфизма между любыми несчётными польскими пространствами (в частности, такими являются все $\mathbb{R}^n$).
Аноним 07/12/15 Пнд 11:42:20 #419 №316419 
>>316413
вот это интересно, можно подробнее?
>>316385>>316387
этим господам тоже спасибо, правда я пока слишком тупой, чтобы понять их содержание

Может ещё какие теоремы есть из функана?
knuebok 07/12/15 Пнд 12:04:33 #420 №316424 
>>316364

Даже я ебал твой мать.
Аноним 07/12/15 Пнд 12:05:50 #421 №316426 
>>316382

Оба учебника говно. Лучше Кудрявцева почитай, толково пишет.
Аноним 07/12/15 Пнд 15:26:28 #422 №316498 
>>316426
Яро вдругорядь реку я: учитнитесеся об Камынина. Камынин весьми красив статен толст.
Аноним 07/12/15 Пнд 17:09:35 #423 №316530 
14494973758840.png
Анон, спаси, а то я сейчас ебу дам... Пишу одну штуку, и дико затупил на элементарной херне. Вся суть - пикрелейтед. Есть два угла A и B, есть два угла C и D которые являются углами отражения, если луч с углом A отражается от плоскости с углом B (угол D) и если луч с углом B отражается от плоскости с углом A (угол C). Нужно подобрать такую функцию, что f(A,B) = D и f(B,A) = C. Вроде хуита, но уже целый день не могу подобрать эту функцию, видимо совсем отупел.
Аноним 07/12/15 Пнд 17:11:30 #424 №316531 
>>316530
Блядь, не углом отражения а серединным углом.
Аноним 07/12/15 Пнд 17:33:18 #425 №316537 
>>316531
Что бы было понятнее что это вообще
(A+B)/2+45' = C или D
Аноним 07/12/15 Пнд 18:54:12 #426 №316574 
Господа, где можно найти формулировку и доказательство теоремы о ядре следа в конечных полях?
Аноним 07/12/15 Пнд 19:01:52 #427 №316577 
>>316419
>вот это интересно, можно подробнее?
Чуть подробнее. Можно интересоваться не собственно топологией, а борелевскими сигма алгебрами, которые из неё получаются (наименьшее семейство подмножеств пространства, содержащее все открытые и замкнутое относительно операций дополнения и объединения счётных семейств). Оказывается, что несмотря на разнообразие топологий, для большой класс пространств является борелевски изоморфными (т.е. существую биекции такие, что образы и прообразы борелевских множеств борелевские). Подробнее написано, например, здесь http://ncatlab.org/nlab/show/Polish+space
Аноним 07/12/15 Пнд 20:31:33 #428 №316586 
Помогите решить следующую задачу. n точек случайно выбираются на окружности (распределение равномерное). Какова вероятность того, что они все будут в одной полуокружности?

Я думаю, надо для каждого диаметра найти эту вероятность и проинтегрировать по полуокружности. То есть проведем диаметр под углом alpha. Вероятность того, что все точки будут по первую сторону от диаметра равна (1/2)^n. По вторую сторону тоже. Чтобы получить искомую вероятность надо как-то проинтегрировать.
То есть получается что-то типа интеграла (1/2)^(n-1) d alpha от 0 до pi.

Я не уверен в подробностях, потому что я дебил-погромист. Че гуглить, чтобы научиться это решать и строго объяснять решение?
Аноним 07/12/15 Пнд 21:23:30 #429 №316589 
>>316586
50/50
Лбо попадут либо не опадут.
Аноним 07/12/15 Пнд 21:29:15 #430 №316590 
14495129552440.png
>>316530
СУУУУКАААА. Как же долго я ебался с этой формулой. Спасибо, наукач, ты как всегда выручаешь в сложный момент.
Аноним 07/12/15 Пнд 21:49:14 #431 №316593 
Анон, где можно скачать пакет комп. алгебры Guava? Насколько он зашкварен?
Аноним 07/12/15 Пнд 22:18:14 #432 №316610 
>>316589
Когда я слушал курс по терверу, нам препод эту шутку рассказал 8435767389678 раз и она даже в 1 раз не была смешной.
Аноним 07/12/15 Пнд 23:44:21 #433 №316634 
>>316610
Ну тут сам понимаешь, 50 на 50.
Аноним 07/12/15 Пнд 23:46:02 #434 №316635 
14495211629340.png
Сап, гуаваёбы. Есть одна программа, и она для второй версии пакета. При вызове функции АффинПоинтсОнКурв возникает ошибка. Однако здесь пример почти такой же:

http://www.gap-system.org/Manuals/pkg/guava3.11/doc/chap5.html#X857EFE567C05C981

Кто-то занимался в этой системе? Каким образом можно разобраться в причине ошибки и пофиксить код?

В релизах только четвертая версия. http://www.gap-system.org/Releases/index.html
Аноним 07/12/15 Пнд 23:48:29 #435 №316637 
14495213094570.png
>>316635
Пример булшита.
Аноним 07/12/15 Пнд 23:53:10 #436 №316638 
>>316634
То есть вероятность, по-твоему, от n не зависит?
Аноним 08/12/15 Втр 00:08:37 #437 №316641 
Есть хоть одна подробная книга по тензорному исчислению ? Не могу найти, хоть на британском хоть на русском .
Аноним 08/12/15 Втр 01:06:30 #438 №316648 
14495259907940.png
>>316586
Спасибо за интересную задачу, ебался с ней пару часов. Пробовал и через условные вероятности, но самое простое в итоге - через вариационный ряд.
Пусть X_1,X_2,...,X_n - независимые равномерно распределённые на [0,1]
Тогда X_(1),X_(2),...,X_(n) - их вариационный ряд
Событие, когда все точки лежат на в одной полуокружности, это такое событие, когда
max( X_(2)-X_(1), X_(3)-X_(2), ..., X_(n)-X_(n-1), 1+X_(1)-X(n) ) >= 1/2
Аноним 08/12/15 Втр 01:39:56 #439 №316651 
>>316638
По-моему и не должна зависеть. Но я условие не читал.
Аноним 08/12/15 Втр 03:51:53 #440 №316656 
>>316648
тащемта щас попробовал решить через условные вероятности, какая-то хуита получается реккурентная.

xi_k - длина минимального куска окружности, соединяющего все точки X1,...,Xk (с нормировкой 1)
Тогда
Prob( xi_{k+1} <= t | \xi_k ) = 1{ \xi_k <= t } (2t - \xi_k)
Поэтому
Prob( xi_{k+1} <= t ) = \int_0^t (2t-x) dF_k(x)
хер знает, как отсюда в общем виде решение вывести.
Аноним 08/12/15 Втр 09:16:52 #441 №316666 
>>316574

Здесь только петухи, какие господа.
Аноним 08/12/15 Втр 11:31:41 #442 №316672 
>>316666
Петух здесь только один.
Аноним 08/12/15 Втр 11:40:10 #443 №316675 
>>316656
отмена, это решение неправильное ёбаный покос
Аноним 08/12/15 Втр 16:53:08 #444 №316739 
Как нужно думать, чтобы научиться делать всякие красивые алгебраические трюки? Поясню на примере.

Пусть S - линейный оператор на конечномерном пространстве V с положительно определенным скалярным произведением <, >. Нужно доказать, что если S - изометрия, то SжS = idV, где Sж - сопряженный к S оператор. Уже известен тот факт, что если S - изометрия, то <Su, Sv> = <u, v> для любых u, v из V.

Вот доказательство из учебника:
Для любых u, v:
<(SжS - idV)u, v> = <(SжS)u, v> - <u, v> = <Su, Sv> - <u, v> = 0.
Тогда
<(SжS - idV)u, (SжS - idV)u> = ||(SжS - idV)u||2 = 0
для любых u, то есть SжS - idV = 0.

Я тоже доказал это утверждене, перед тем как прочитать доказательство в учебнике, но мое доказательство заметно длиннее. Как научиться такие пиздатые трюки делать? Как конкретно можно было догадаться, что нужно рассмотреть <(SжS - idV)u, v>?. Вообще в алгебре я очень часто такие трюки вижу.
Аноним 08/12/15 Втр 18:22:02 #445 №316755 
>>316739
Смекалочка
knuebok 08/12/15 Втр 19:01:01 #446 №316759 
>>316739
Зачем какие-то id-ы.
<S-star S u, v> = (определение сопряженного оператора) <S u, S v> = (определение изометрии) <u,v>
Если для всех u,v <Au,v> = <Bu,v> то A=B, следовательно S-star S = Id.
knuebok 08/12/15 Втр 19:02:41 #447 №316760 
Догадаться, что нужно использовать определение S-star в задаче, где есть S-star очень просто, кстати, не вижу в этом никакой смекалочки.
Аноним 08/12/15 Втр 19:21:01 #448 №316767 
14495916617690.png
>>316760
А, ну я проебался, не всю ситуацию рассказал. Изометрия в книге определяется как отображение, которое сохраняет норму, есть теорема пикрелейтед и она доказывается так:
a => b => c => d => e => a.
То есть это автор так доказывал b => c (как будто он не знает, что b => a).

Согласен, что намного удобнее заметить, что b => a (это вообще очевидно) и применить рассуждения
>>316759

Просто, видимо, автор захотел выебнуться хитрым трюком.
Аноним 08/12/15 Втр 22:02:11 #449 №316786 
14496013313960.png
Помогите дурачку.
>имеет в этой окрестности производные до (n−1) порядка включительно
Вот как определить, производную до какого порядка имеет функция?
Аноним 08/12/15 Втр 22:04:08 #450 №316788 
>>316786
Берёшь и без задней мысли дифференцируешь, пока получается. Главное - не забыть перед этим накатить картофана.
knuebok 08/12/15 Втр 22:05:58 #451 №316789 
>>316786
А как определить, что функция имеет производную вообще?
Аноним 08/12/15 Втр 22:11:24 #452 №316792 
>>316789
Если функция дифференцируема в точке, то она имеет производную
knuebok 08/12/15 Втр 22:13:11 #453 №316793 
>>316792
Ну вот, а если функция имеет производные до n-го порядка в точке, то она имеет производную производной производной ... (n раз) ... производной.
Аноним 08/12/15 Втр 22:15:53 #454 №316794 
>>316788
Типа если получается то же самое, то всё пиздец?
Аноним 08/12/15 Втр 22:55:37 #455 №316797 
14496045372050.png
>>316635
Guava 3.12
GAP 4.7.8
Аноним 08/12/15 Втр 23:03:20 #456 №316798 
14496050000750.png
Наукач,прошу,объясни как это решить
к=1
Аноним 08/12/15 Втр 23:06:30 #457 №316799 
>>316637
Дак ты гуаву не подключил!
LoadPackage("guava");
Аноним 08/12/15 Втр 23:23:56 #458 №316800 DELETED
>>316798
В /un/, быдлоэкономист.
Аноним 08/12/15 Втр 23:53:45 #459 №316803 
Бля, кароч есть одна ф-я x^2+6x=0, надо описать и построить. Оно вроде и пиздец легко кажется, только я тупенький совсем. Помогите плз. Я даже разобраться каноническим уравнением не могу тут.
Аноним 08/12/15 Втр 23:54:23 #460 №316804 
>>316803
БЛЯДЬ! ТАМ Y! 6Y!
Аноним 09/12/15 Срд 00:45:41 #461 №316810 
>>316804
http://www.wolframalpha.com/input/?i=x%5E2%2B6y%3D0
is it u Философ-кун 09/12/15 Срд 01:05:36 #462 №316816 
Сижу и дрочу на анона-математика. По-моему, это умилительно. Давайте так: раз-два-два-три-четыре.... И не пять...! Три-четыре? Пять! Еб вашу мать! Спиздили блять. Суки.
Аноним 09/12/15 Срд 05:19:42 #463 №316823 
14496275820600.png
>>314306 (OP)
Математики, извините, если спрашиваю слишком много, но у меня пикрилейтед.
Я пролежал приличную долю семестра в больничке и теперь пытаюсь нагнать упущенное, однако осилить Фурье не могу. Четвёртое задание я сам решил, но мне нужно его как-то проверить, а для этого мне нужен чужое решение, чтобы их сравнить.
Помогите плиз, желательно чтобы Фурье был с объяснением действий, чтобы было легче разобраться.
Аноним 09/12/15 Срд 06:01:07 #464 №316827 
>>316823
Спроси на math.hashcode
Прямо напиши в общих чертах что ты уже решил и спроси правильно ли, и как вообще решать следующий
Чтобы аккуратно писать там формулы можно использовать латех 
Вот в этом онлайн редакторе есть все символы и значечки
https://www.codecogs.com/latex/eqneditor.php
набиваешь там формулы, копируешь и вставляешь в свой вопрос обернув слева и справа значками $$
Тут кто-то короткий фак для нубов делал, включите туда эту инструкцию
можете подредактировать
Аноним 09/12/15 Срд 09:12:56 #465 №316829 
>>316759
>Зачем какие-то id-ы
Чтобы доказать вот это
>Если для всех u,v <Au,v> = <Bu,v> то A=B
Ибо надо вычесть и посмотреть на скалярный квадрат.
Аноним 09/12/15 Срд 13:50:46 #466 №316849 
А какому универсальному свойству отвечает полупрямое произведение групп? Зачем оно вообще нужно? Можно ли "увидеть", что какая-то группа есть полупрямое произведение, по ёё действию на чём-то например?
Аноним 09/12/15 Срд 14:37:47 #467 №316863 
>>316586
УУ, СУКА Я ВСЁ ТАКИ ЕЁ РЕШИЛ ЗАДАЧУ ЭТИМ СПОСОБОМ >>316648, НО ЕБАЛСЯ 3 ДНЯ
способ с условными вероятностями, кстати, нихуя не правильный
Ответ у меня получился n/2^n.
Ещё раз спасибо за задачу, положу её в копилку.
Аноним 09/12/15 Срд 15:28:40 #468 №316874 
>>316863
То есть для 1 точки ответ 1/2? Предположим, что эта формула верна для n >= 3. Очевидно, что для 3х точек ответ не 3/8. Возьмем любую из трех точек и проведем через нее диаметр. Вероятность того, что оставшиеся 2 точки будут по одну сторону от проведенного диаметра, равна 1/2. А искомая вероятность не меньше этой.
Аноним 09/12/15 Срд 15:29:46 #469 №316875 
>>316874
вроде обсчитался, 2n/2^n
knuebok 09/12/15 Срд 15:33:40 #470 №316877 
>>316829
Лил, если зафиксирован базис, то достаточно заметить, что <A e_j, e_i> - это элемент на месте (ij) в матрице оператора А.
knuebok 09/12/15 Срд 16:55:39 #471 №316893 
>>316877
Ортонормированный, естественно.
Аноним 09/12/15 Срд 16:55:56 #472 №316894 
14496693565700.png
Опа, математики тред, я в правильном месте. Потребовалось разделить 101 на 500, ну я начал столбиком получилось 0.2002, потом проверил умножением и нехуя не вышло правильновизуально я понял что должно быть 0.202, потом добрался до калькулятора, а он говорит что 0.202. И я чёт не могу нагулить как правильно делить в столбец такие числа. Что я делаю не так и почему? Ебать, день о котором меня предупреждали настал, потребовалось поделить столбиком, а я не смог. Я так понял, что зря 0 приписал, но вроде же типа добавили 0 к 100 что бы 100 стало 1000 и ноль приписали к искомому числу.... Ну сначало к 10 приписал, стало 100)опять не делится) и вышло 0.20, потом к 100 приписал 0.... чё такое?
Аноним 09/12/15 Срд 17:17:57 #473 №316897 
Решил заняться математикой для себя, т.к. в вузе с ней все тут плохо. Наткнулся на книги группы Бурбаки. Они еще актуальны?
Аноним 09/12/15 Срд 17:23:43 #474 №316898 
>>316897
Они актуальны, но учиться по ним не стоит
Аноним 09/12/15 Срд 17:24:04 #475 №316899 
>>316897
именно с них и надо начинать
Аноним 09/12/15 Срд 17:27:11 #476 №316900 
>>316899
Точно с них?
У меня друг с матфака ВШЭ хорошо разбирается во всяких топологиях, группах и т.п., но учился по современной литературе.
Или в Бурбаки все абсолютно фундаментально и полезно? Просто я хочу добиться как можно более глубокого чувства каждого раздела и связи между этими разделами, чтобы как можно свободнее математически мыслить, ну ты понял.
knuebok 09/12/15 Срд 17:31:25 #477 №316901 
>>316900
Учись по современной литературе, тот кун тебя траллирует.
Аноним 09/12/15 Срд 17:33:10 #478 №316902 
>>316901
А есть какой-то FAQ по авторам и разделам?
Аноним 09/12/15 Срд 17:37:51 #479 №316903 
14496718717650.png
Ох я и проигрываю с такого аутизма.

Вообще не люблю задачи, почему в каждом тексте нужно обязательно дристануть этими задачками ебаными. Только время на них зря тратить. Некоторые совсем ебанутые уже их в научные статьи суют.

Алсо, может посоветуете годноты по теории Галуа. Но только чтобы там было без гротендиков и пучков, или хотя бы не сразу.
Аноним 09/12/15 Срд 17:42:43 #480 №316904 
>>316903
Ну это сделано для того, чтобы не засорять текст рутинными и очевидными доказательствами.
knuebok 09/12/15 Срд 17:58:42 #481 №316905 
>>316902
Ну есть рекомендации Вербита по программе матфака http://verbit.ru/Job/HSE/Curriculum/all.txt например, учебнике, по мне, там порекомендованы ничёшные (но я бы кое-что поменял).
knuebok 09/12/15 Срд 17:58:58 #482 №316906 
*учебники
Аноним 09/12/15 Срд 18:24:36 #483 №316909 
>>316902
Вот я сейчас читаю и решаю Munkres "Topology", вообще годно. Чтобы начать читать, не нужно знать вообще ничего, даже теория множеств там дается в первой главе.
Аноним 09/12/15 Срд 18:29:52 #484 №316911 
14496749927600.png
А почему тут такая бурбакофобия, а? Я нигде не видел определения функции лучше, чем у бурбаков, например.
Аноним 09/12/15 Срд 18:34:10 #485 №316912 
>>316911
И тут тыт такой даешь определение функции по бурбакову.
Аноним 09/12/15 Срд 18:43:10 #486 №316916 
14496757905160.png
14496757905181.png
>>316912
Там кроме собственно функции еще дается определение пары и графика (на которых основано определение функции). Ну и как подмножество декартова произвеения множеств функция тоже разбирается очень подробно.
sageАноним 09/12/15 Срд 18:48:10 #487 №316917 
>>316911
Мне показалось трудно читать человеку с низким левелом математической культуры
даже матшкольнику-младшекурснику, и то сложно
Для более "взрослых" может и ок.
Аноним 09/12/15 Срд 18:51:30 #488 №316918 
14496762904490.png
>>316909
>даже теория множеств там дается в первой главе.
Да любой автор считает своим долгом в первых главах обсосать теорию множеств и определение функции, даже если в дальнейшем требуется знание ещё кучи материала чтобы читать основной текст учебника.
Аноним 09/12/15 Срд 18:51:36 #489 №316919 
>>316916
Своими словами написать не можешь значит идешь в деревенский туалет типа сортир, жрать говно ведрами.
Аноним 09/12/15 Срд 18:53:42 #490 №316921 
>>316917
>матшкольнику-младшекурснику, и то сложно
Разве? Если матшкольнику давали теорию и сведения об основаниях математики то должно быть норм.
Аноним 09/12/15 Срд 18:54:06 #491 №316922 
>>316919
Своими словами, функция - это график пар.
Аноним 09/12/15 Срд 19:36:36 #492 №316935 
>>316922
Множество пар где для левого элемента есть единственный правый.
Аноним 09/12/15 Срд 20:10:25 #493 №316945 
>>316799
>>316797
Спасибо! Похоже, охуительный пакет для алгеброгеометрической теории кодирования. Есть где-нибудь кратное введение в основы гуавы? Например, почему где-то требуется ставить две точки с запятой, а где-то хватает одной?

Алсо, реквестирую теорему о ядре следа в конечных полях. Та, что о количестве элементов в ядре. Нигде не могу найти.
Аноним 09/12/15 Срд 21:43:17 #494 №316956 
>>316945
guava - отдельный пакет в GAP для всяких криптографий и наоколо (читай: библиотека). GAP - открытая оболочка, для которой все подряд пишут разные интересные математические тулзы. Другие системы компьютерной алгебры часто подключают GAP со всеми его пакетами, так как он достаточно функционален (точно знаю про Magma и Sage; вроде Maxima еще). На сайте есть подробная документация; как ты уже видел, по отдельным пакетам есть описания функций, которые туда входят. Возможно, по разным конкретным вопросах можно легко найти всякие гайды и книжечки.
Каждый вызов функции заканчивается ";". Двойная ";;" не будет выдавать результат в консоле (посмотри разницу на тех же скринах сверху).
След - линейное отображение большого поля НА маленькое, если смотреть на них как на векторные пространства (или как на абелевые группы даже). Далее вспоминаем первую теорему о гомоморфизме.
Аноним 09/12/15 Срд 22:04:12 #495 №316961 
Сап, математики, посоветуйте годную и понятную книгу по мат.физике, пожалуйста.
knuebok 09/12/15 Срд 22:26:54 #496 №316963 
>>316961
Двачую реквест.
Аноним 09/12/15 Срд 22:31:07 #497 №316964 
>>316961
Владимиров
Аноним 09/12/15 Срд 22:34:44 #498 №316965 
>>316964
спасибо, выглядит годно, пошел погружаться.
Аноним 10/12/15 Чтв 00:40:26 #499 №316982 
Как по-русски называется вот эта теорема?
https://en.wikipedia.org/wiki/Rank%E2%80%93nullity_theorem
Аноним 10/12/15 Чтв 01:34:35 #500 №316990 
>>316982
Конкретно эта- не знаю, но это простое следствие теоремы о гомоморфизме.
Аноним 10/12/15 Чтв 01:52:59 #501 №316993 
>>316961
Evans
Аноним 10/12/15 Чтв 02:22:43 #502 №316995 
>>316877
Так там суть в том, что все без упоминания базисов. Хорошая книжка, наверное.
Иначе можно просто перемножить А и А^t и не писать каких-либо теорем и доказательств.
Аноним 10/12/15 Чтв 09:40:46 #503 №317008 
14497296462330.png
Где прочитать про решение пикрелейтеда?
А то в предложенном преподшей вроде не все есть, да и написано криво.
Аноним 10/12/15 Чтв 10:20:54 #504 №317012 
>>316916

Это не определение, просто понятие функции исходное и неопределяемое (как и N). Пара по сути это и есть выражение соответствия и наоброт.
Аноним 10/12/15 Чтв 10:40:04 #505 №317013 
>>317012
Неопределяемых понятий не бывает. Бывают технические термины - просто набор символов, который можно без каких-либо потерь заменить на любой другой нигде не задействованный набор символов. В технических терминах нет смысла, они ничего не значат сами по себе, это просто бессмыслица, никакого "интуитивного понимания" они не имеют. Когда мы накладываем на технические термины какие-то синтаксические ограничения, мы тем самым определяем некоторое понятие.

Идею, что есть какие-то неопределяемые понятия, придумали ебанутые на всю голову, безумные, алогичные школьные учителя.

>>317008
Первое очевидно, A=B. Остальное Игошин.
Аноним 10/12/15 Чтв 13:23:34 #506 №317038 
N-петух станет теперь ещё и f-петухом? Одобряю.
Аноним 10/12/15 Чтв 14:58:03 #507 №317058 
14497486835290.png
>>317012
>понятие функции исходное и неопределяемое (как и N).
Ну что же ты так, братишка. Ладно N, теперь еще и функция. Что там еще неопределяемое, чтобы в картинку добавить?
>Пара по сути это и есть выражение соответствия и наоброт.
Ну. Соответствие. Почему это не определение, если как раз-таки однозначное определение?
Аноним 10/12/15 Чтв 22:48:03 #508 №317087 
С чего начать изучение математики? В школе и вузе знал её отлично, сейчас уже ничего не помню, есть только куча хаотичных-полузабытых знаний.
Аноним 11/12/15 Птн 10:00:07 #509 №317093 
>>317087

С определения N и f.
Аноним 11/12/15 Птн 12:16:06 #510 №317122 
>>317093
N петушок как всегда незаметен.

Я вот учу производные и мне пишут про вычисление формулы касательной графика функции в точке.
Собственно, зачем нужна эта касательная прямая? Если функция периодическая, касательная невозможна?
Перекатs Аноним 11/12/15 Птн 14:53:31 #511 №317158 
>>317156 (OP)
>>317156 (OP)
>>317156 (OP)
comments powered by Disqus

Отзывы и предложения